Kaplan MSK

Lakukan tugas rumah & ujian kamu dengan baik sekarang menggunakan Quizwiz!

A 66-year-old woman picks up a bag of groceries out of the supermarket cart to place it in the trunk of her car. As she does so, she feels sharp, sudden pain in the middle of her arm, and her humerus suddenly breaks. She arrives at the emergency department cradling her arm; the deformity leaves no doubt that the bone is broken. She is unaware of any underlying medical conditions. Which of the following is the most likely reason for the fracture? A. Bony metastasis to the humerus from breast cancer B. Osteitis fibrosa cystica from parathyroid disease C. Osteomalacia from nutritional deficiency D. Osteoporosis E. Primary malignant bone tumor

The correct answer is A. A fracture from such trivial strain signifies a very weakened bone. In this age and gender, the most likely cause would be a lytic lesion from metastatic breast cancer. In a man, we would have suspected metastatic lung cancer (not prostate, because prostatic metastases are blastic rather than lytic). Clinical Pearls Pathologic fracture is a fracture that occurs through an abnormal bone matrix. The most common pathologic fractures are secondary to metastatic or primary bone cancer and osteoporotic type of fracture. The cancers that are most likely to result in bone metastasis are prostate, thyroid, breast, lung, and kidney.

An 8-year-old boy has left leg pain and a high fever, and has been refusing to walk since yesterday. On physical examination, his temperature is 39.8ºC (103.6ºF), blood pressure 122/68 mm Hg, pulse 102/min, and respirations 20/min. The left femur is tender to palpation 3 cm above the left knee and there is marked soft-tissue swelling. Radiograph of the left leg is normal. Bone scan shows increased uptake around the metaphysis of the left femur. Which of the following is the most likely pathogen? A. Escherichia coli B. Haemophilus influenzae C. Salmonella D. Staphylococcus aureus E. Streptococcus pneumoniae

The correct answer is D. Osteomyelitis is a pyogenic infection of the bone. Pathogenesis is similar to septic arthritis, with the origin of infection occurring from hematogenous spread, direct extension of a local infection, or direct inoculation of bone either from trauma or surgical manipulation. In children the most frequent presentation is acute hematogenous spread. The most common location of osteomyelitis is the metaphysis of the distal femur and proximal tibia. The most prevalent pathogens are the same as those seen in septic arthritis. Staphylococcus aureus is the most common pathogen followed by group A beta-hemolytic streptococci. Neonates are at risk for group B beta-hemolytic streptococci, but risk is dramatically lower since pregnant women are routinely tested and treated for this pathogen prior to labor. Most patients who have osteomyelitis will present with a chief complaint of fever and bone pain. The pain is usually severe, constant, and aggravated by motion. The older the child, the more exquisite the point tenderness. That is because the bone has a thicker metaphyseal cortex with a dense fibrous periosteum. Localized swelling, warmth, and erythema are signs seen late in the infection, as the periosteum becomes more involved. Neonates can present with vague symptoms, consisting only of irritability and poor feeding, or they can show signs of fulminant sepsis. Peripheral WBC count may be normal or elevated with a left shift with a predominance of neutrophils. Erythrocyte sedimentation rate (ESR) is usually elevated and blood cultures are positive in approximately 60% cases. Bone cultures taken either surgically or by needle aspiration result in a culture yield of 80%. Plain radiographs may be normal for up to 2 weeks from the onset of illness, and the earliest signs on plain radiographs are soft-tissue swelling and displacement of muscle plane. Bony changes begin to appear by 7-10 days, starting with a hazy appearance of the metaphysis followed by irregular areas of trabecular necrosis and absorption. Eventually subperiosteal new bone formation occurs as the infection spreads through the cortex. A bone scan usually diagnoses osteomyelitis as early as 24-48 hours from onset. Clinical Pearls Osteomyelitis is an infection of bone that is usually bacterial in origin. Microorganisms can be introduced into bone in three ways: Hematogenous delivery Direct inoculation (usually traumatic, but also surgical) Local invasion from a contiguous infection (usually decubitus ulcer or periodontal disease) In children, acute osteomyelitis is primarily hematogenous in origin. Risk factors for hematogenous osteomyelitis in children include the following: Sickle cell disease Immunodeficiency disorders, such as chronic granulomatous disease Sepsis Minor trauma coincident with bacteremia Indwelling vascular catheters Chronic hemodialysis vascular access The growing metaphyses of long bones are most commonly involved, particularly the femur, tibia, or fibula. Bacteria, principally Staphylococcus aureus, cause the vast majority of cases of acute osteomyelitis in children. Staph aureus is the most common pathogen for children with osteomyelitis. Among patients with sickle cell disease (SCD), hematogenous osteomyelitis is most often caused by Salmonella or other gram-negative organisms, such as E. coli.

A 37-year-old woman presents with a swollen and tender left wrist. The symptoms began approximately a day earlier and have become increasingly severe. She has noted a fever and shaking chills last night. She has no past medical history and takes no medications, except for ibuprofen for occasional premenstrual cramps. On physical examination the wrist has decreased ROM and is tender to palpation. The joint space has an obvious effusion and is tender, erythematous, and warm. Which of the following is the most appropriate next step in management? A. Colchicine B. Indomethacin C. IV antibiotics D. Wrist radiograph E. Arthrocentesis

The correct answer is E. A monoarticular arthritis should raise the question of a septic versus a crystalline arthritis. Septic arthritis and crystalline arthritis can be notoriously similar in their clinical presentations, so clearly establishing the diagnosis, particularly in a patient with no known history of crystalline disease, is of great advantage before initiating definitive therapy. Crystal-induced and septic-induced joint can coexist in the same joint. An arthrocentesis is the most appropriate choice and allows examination of fluid for WBCs, crystals, and bacteria. Clinical Pearls Any patient who has a hot, swollen, inflamed, and motion-restricted joint should have aspiration performed to help determine the diagnosis. If this joint is a prosthetic joint, this aspiration should be performed in the operating room to perform this aspiration under aseptic conditions. American College of Rheumatology guidelines for a patient with monoarticular arthritis after completion of a thorough history and physical examination: Patients who have a history of significant trauma or focal bone pain should have plain radiographs of the affected joint to rule out fracture, tumor, or metabolic bone disease. In the absence of a history of trauma or following a radiograph which excludes fracture or dislocation, an effusion or other signs of inflammation are markers of infection until proven otherwise. Joint aspiration is therefore the next diagnostic step. A bloody effusion should lead to consideration of a coagulopathy, pseudogout, tumor, trauma, or a Charcot joint; subsequent evaluation includes a PT, PTT, platelet count, and bleeding time. Bone marrow elements present in the synovial fluid are suggestive of an intra-articular fracture. A noninflammatory synovial fluid (e.g., <2,000 WBCs or <75% neutrophils) should lead to consideration of osteoarthritis, soft-tissue injury, or viral infection. Inflammatory joint fluid with crystals establishes the diagnosis of gout or pseudogout. A positive synovial fluid culture establishes the diagnosis of infectious arthritis. A sterile inflammatory joint fluid raises the suspicion of systemic rheumatic disorders; such patients should have further evaluation that may include a CBC, ESR, RF, anti-CCP, liver function tests, HLA-B27, ANA, or Lyme serologies.

A 23-year-old woman wearing high-heeled shoes has an inversion injury to her ankle and sprains her ankle while running down a flight of stairs. Which of the following ligaments did she most likely injure? A. Anterior talotibial B. Anterior talofibular C. Calcaneotibial D. Deltoid E. Medial collateral

A 23-year-old woman wearing high-heeled shoes has an inversion injury to her ankle and sprains her ankle while running down a flight of stairs. Which of the following ligaments did she most likely injure? A. Anterior talotibial B. Anterior talofibular C. Calcaneotibial D. Deltoid E. Medial collateral Show/hide explanation The correct answer is B. The most common type of ankle sprain is lateral, which occurs as a result of excessive inversion of the foot and dorsiflexion of the ankle, which causes injury to the anterior talofibular ligament. The anterior talofibular (not calcaneotibial, choice C; not talotibial, choice A) ligament may tear, producing marked swelling and pain. These 2 ligaments, combined with the posterior talofibular ligament, constitute the lateral ligament of the ankle. Clinical Pearls Ankle injuries are among the most common injuries seen by clinicians in primary care offices and emergency departments. The most common mechanism of ankle injury is inversion of the plantar flexed foot, which causes damage to the lateral ligaments of the ankle. This ligament complex consists of the anterior talofibular ligament, the calcaneofibular ligament, and the posterior talofibular ligament The anterior talofibular ligament is the first or the only ligament to be injured in the majority of ankle sprains. The most commonly injured ligament is the anterior talofibular ligament, the second is the calcaneofibular ligament, and the third is the posterior talofibular ligament (this type of injury occurs because of ankle inversion).

A football player is examined by the team's health care provider following a shoulder injury during a game. Preliminary radiographs show an inferior dislocation of the humerus. On further examination a weakness in lateral rotation and abduction of the arm is noted. Which of the following nerves is most likely to be injured? A. Axillary B. Dorsal scapular C. Radial D. Suprascapular E. Thoracodorsal

The correct answer is A. Because of the proximity of the axillary nerve to the glenohumeral joint, a fracture of the surgical neck of the humerus or an inferior dislocation of the humerus could damage the nerve. The axillary nerve innervates the deltoid muscle. The deltoid abducts, adducts, flexes, extends, and rotates the arm medially. The axillary nerve also innervatesthe teres minor, which rotates the arm laterally. Clinical Pearls Shoulder dislocations account for 50 percent of all major joint dislocations. Anterior dislocation composes 95 to 97 percent of cases; posterior dislocation, 2 to 4 percent; inferior dislocation, 0.5 percent. Inferior dislocations are commonly caused by axial loading with the arm fully abducted or forcefully hyperabducted. This dislocation frequently occurs when patients fall and suddenly grasp on to an object above their head resulting in hyperabduction. Patients with this injury hold the involved arm above their head and are unable to adduct the arm. The forearm is pronated and in most cases rests on the top of the head. The axillary nerve is most commonly injured with inferior shoulder dislocations. Arterial injury may also occur. Radiographs reveal the humeral head beneath the coracoid or the glenoid with inferior dislocations. Associated fractures include the greater tuberosity (most common), acromion, scapula, humeral head, coracoid, and glenoid.

A 46-year-old woman visits her podiatrist to have several bunions removed from her right foot. She chooses conscious sedation rather than general anesthesia for this procedure. She is given IV midazolam (Versed) to supplement the local anesthetics that are injected into her foot. Midway through the surgery, she suddenly becomes agitated and combative and exhibits involuntary movements. The anesthesiologist immediately administers which of the following? A. Flumazenil B. Glucagon C. Naloxone D. Nitrite E. Protamine

The correct answer is A. Flumazenil is a benzodiazepine antagonist and has been approved to hasten the recovery from benzodiazepines used in anesthetic and diagnostic settings and to reverse the CNS depressant effects following an overdose with benzodiazepines. Flumazenil can be used only for benzodiazepines; it is not useful in reversing the effects of other CNS depressants, such as barbiturates and ethanol.

A patient who has severe systemic lupus erythematosus is receiving long-term glucocorticoid therapy. She should consequently receive supplemental therapy with which of the following? A. Calcium B. Carotene C. Folate D. Iron E. Vitamin B12

The correct answer is A. Long-term corticosteroid use can cause osteopenia with vertebral compression factors. This can be minimized with oral calcium supplementation, vitamin D, and bisphosphonates. Exercise in moderation may stimulate bone formation. The most commonly used treatment for osteopenia and osteoporosis are bisphosphonates, which require that patients have sufficient intake of calcium and vitamin D. Clinical Pearls Corticosteroids are often used in the management of SLE. Complications of glucocorticoids include osteoporosis, weight gain, aseptic necrosis, Cushing syndrome, infection, hypertension, hyperglycemia, and cataracts. With regard to osteoporosis, bone loss is most significant with the first 6 months of treatment with corticosteroids. To prevent this bone loss, calcium and vitamin D and bisphosphonates are utilized.

A 35-year-old man falls on his outstretched hand and comes to the urgent care clinic complaining of wrist pain. He relates that he was not able to break the fall, and that the heel of his hand took the brunt of his full weight as it hit the pavement. On physical examination, he is distinctly tender to palpation over the anatomic snuff box. Anteroposterior and lateral radiographs are negative. Which of the following are the most likely diagnosis and most appropriate next step in management? A. Carpal scaphoid fracture; thumb spica cast B. De Quervain tenosynovitis; steroid injections C. Displaced scaphoid fracture; open reduction and internal fixation D. Ligamentous injury; Ace bandage and analgesics E. No fracture; reassurance

The correct answer is A. Nondisplaced fractures of the carpal scaphoid bone are notorious for not showing up on radiographs at the time of injury. The mechanism of injury and the physical findings described in this vignette are sufficient to make a presumptive diagnosis and to indicate the use of a thumb spica cast. Tenderness at the anatomical snuffbox in the setting of a patient sustaining a fall on outstretched hand provides enough support for applying a thumb spica cast for presumed scaphoid fracture regardless of what the initial x-rays show. Clinical Pearls Nondisplaced fractures of the carpal scaphoid bone are notorious for not showing up on radiographs at the time of injury. A thumb spica cast is indicated for treatment; there is no need for open reduction for nondisplaced fractures.

A football player is tackled and he develops severe knee swelling and pain. On physical examination with the knee flexed at 90 degrees, the leg can be pulled anteriorly, like a drawer being opened. There is no clear end point with this maneuver. A similar finding can be elicited with the knee flexed at 20 degrees by grasping the thigh with one hand, and pulling the leg with the other hand.Which of the following is the most likely injured structure? A. Anterior cruciate ligament B. Lateral collateral ligament C. Medial collateral ligament D. Medial meniscus E. Posterior cruciate ligament

The correct answer is A. Swelling of the knee after trauma usually denotes the presence of a significant injury. The tests described (anterior drawer and Lachman test) are used in order to identify injury to the anterior cruciate ligament. Clinical Pearls The anterior cruciate ligament (ACL) is the most serious and commonly injured knee ligament. Noncontact, low-energy injuries that occur during athletic activity account for the majority of ACL tears. Female athletes are at increased risk due to an increased Q angle because of their wider hips. The typical mechanism for a noncontact ACL injury involves a running or jumping athlete who suddenly decelerates and changes direction (e.g., cutting) or pivots in a way that involves rotation or lateral bending (i.e., valgus stress) of the knee. Patients who sustain an ACL injury often complain of feeling a "pop" in their knee at the time of injury, acute swelling thereafter, and a feeling the knee is unstable or "giving out." Nearly all patients with an acute ACL injury manifest a knee effusion from hemarthrosis. The Lachman and anterior drawer tests are the most useful for detecting ACL injury.

A football player suffers an anterior dislocation of the shoulder. Cutaneous sensation over the lower half of the deltoid muscle is impaired. These findings suggest damage to which of the following nerves? A. Axillary B. Median C. Musculocutaneous D. Radial E. Ulnar

The correct answer is A. The axillary nerve can be damaged during anterior dislocation of the shoulder, causing loss of sensation in the skin overlying the lower half of the deltoid muscle. Clinical Pearls Anterior shoulder dislocations are the most common type of shoulder dislocation. The injury is usually caused by a blow to the abducted, externally rotated, extended arm (e.g., blocking a basketball shot). A blow to the posterior humerus or a fall on an outstretched arm may cause an anterior dislocation. An anteriorly dislocated shoulder causes the arm to be slightly abducted and externally rotated. The patient resists all movement. The acromion appears prominent in thin individuals, and there is loss of the normal rounded appearance of the shoulder The axillary nerve is most commonly injured in anterior shoulder dislocations. Axillary nerve dysfunction manifests as loss of sensation in a "shoulder badge" distribution. Deltoid muscle weakness may also be present.

A 46-year-old woman who had always been in good health comes in because of the sudden onset of very severe back and leg pain. She pain began 2 hours ago when attempting to lift a heavy object. She says that she felt "a bolt of lightning" running down the back of her leg, and she still has very severe pain that prevents her from walking or moving. The pain is exacerbated by coughing, sneezing, or straining. She keeps the affected leg flexed; straight leg-raising gives her excruciating pain. She has good anal sphincter tone and intact sensation in the perineum. Once the diagnosis is confirmed with the appropriate studies, which of the following is the most appropriate treatment? A. Analgesics with avoidance of activities that cause pain B. Appropriate antibiotics C. Long-term body cast D. Radiotherapy to the affected area E. Surgical decompression

The correct answer is A. The clinical features are those of a herniated lumbar disc. The diagnosis should be confirmed with an MRI only after 6 weeks of symptoms unless the patient has red flag symptoms. The patient should be treated conservatively with avoidance of activities that bring symptoms along with NSAIDs or acetaminophen. Most patients get better with this simple approach. Giving antibiotics (choice B) assumes an infectious process. Infections can occur in the lumbar spine or the discs, but their symptoms do not start suddenly, like this vignette describes. Patients utilizing illicit IV drugs are at highest risk for the development of vertebral infections. Clinical Pearls The vast majority of patients with simple herniated lumbar discs do not need any imaging because the majority of these patients will improve with conservative care. If the patient fails to respond to conservative care or develops red flag symptoms, herniated lumbar disc is best confirmed by MRI. Surgical intervention is needed if neurologic deficits are progressing. Emergency intervention is required if there is a cauda equina syndrome.

The parents of a 5-year-old boy come to the health care provider concerned about their child's recurrent leg pains. The boy has been complaining for several weeks about pain in both legs, usually occurring soon after going to bed. He derives relief from rubbing his legs and knees. He does not limp and is able to participate in sports activities, such as tag and basketball, with other kids in the neighborhood. Which of the following is the most likely diagnosis? A. "Growing" pains B. Juvenile rheumatoid arthritis C. Osgood-Schlatter disease (osteochondritis of the tibial tubercle) D. Osteoid osteoma E. Osteosarcoma F. Stress fractures

The correct answer is A. The clinical history provided by the parents is consistent with "growing" pains. Although such pains are most likely unrelated to growth, they do affect children between age 3 and 10 years. Growing pains are most commonly bilateral, involve the lower leg and knees, manifest with pain during rest (usually at bedtime), and are relieved by massaging or rubbing. Children awaken the next morning feeling fine. Physical activity is not impaired. Limb pains produced by organic disease will usually be unilateral (except for rheumatoid arthritis) or associated with physical signs (e.g., swelling, warmth). The child who has physical injuries or disease cannot bear to have the affected area touched. Growing pains often have a familial predisposition. Clinical Pearls Recurrent, self-limited extremity pain in which there is no explanation are often called "growing pains." Growing pains are defined as pain awakening a child (at night or from naps) who is otherwise without any manifestation of musculoskeletal problems. Although growing pains are a common musculoskeletal condition, their etiology is not known. It is important that appropriate evaluation is performed to differentiate growing pains from other more serious conditions. Growing pains occur most commonly in preschool and school-aged children. Pain occurs primarily in the lower extremities. Pain is typically bilateral and located deep in the legs, usually the thigh or calf. Pain is paroxysmal and may be severe enough to make the child weep; symptom-free periods of days, weeks, or months occur between episodes. Daytime activities are not affected by growing pains.

A patient presents to the health care provider because of "something wrong with her foot" that causes her to trip and fall. Physical examination reveals an inability to dorsiflex the foot and a loss of sensation between the first and second toes. There is no other motor or sensory loss. Which of the following nerves was most likely injured? A. Deep peroneal B. Femoral C. Superficial peroneal D. Sural E. Tibial

The correct answer is A. The deep peroneal nerve arises from the common peroneal nerve (L4-S2). It innervates the postaxial muscles of the anterior compartment of the leg, which dorsiflex (extend) the foot. Damage to the nerve therefore produces "foot-drop," a classic clue to deep peroneal nerve pathology. The nerve gives rise to many branches, one of which innervates the skin between the first and second toes. Clinical Pearls The most frequent site of injury to the common peroneal nerve is just below the knee as the nerve wraps around the lateral aspect of the fibula and immediately before the nerve divides into its deep and superficial branches. The deep peroneal nerve provides sensory input from the webspace between the first and second toe. The superficial peroneal nerve provides sensory input from the dorsal foot and lateral shin. This dermatome of the leg is innervated by the L4 dermatome. The clinical presentation of common peroneal neuropathy at the fibular neck is usually acute foot drop (difficulty with dorsiflexing the foot against resistance or gravity). Patients may also complain of paresthesias and/or sensory loss over the dorsum of the foot and lateral shin (superficial peroneal nerve territory). Examination typically reveals weakness in foot dorsiflexion and foot eversion (deep and superficial peroneal nerves, respectively), with normal inversion and plantar flexion (posterior tibial nerve). Reflexes are normal. For example, in patients with peroneal neuropathy at the fibular neck, no specific treatment is available other than removing pressure on the nerve (e.g., use extra cushions while sleeping and avoid crossing the legs during the day). An ankle-foot orthosis splint, to keep the foot dorsiflexed, should be used until active movement has recovered. Physical therapy may help until the nerve recovery is complete.

A 53-year-old man comes to the office complaining of severe right toe pain. The patient was awakened in the middle of the night by a sharp, burning pain localized to the first metatarsophalangeal joint. The toe is noted to be warm, painful to touch, and edematous; the patient cannot move the joint in any direction without extreme pain and will not allow the toe to be covered. The patient's medical history is significant for hypertension for which he takes a thiazide diuretic. He consumes about 3 pints of beer per day. Which of the following is most likely to confirm the diagnosis? A. Aspiration of fluid from the affected joint B. Determination of serum urate concentrations C. Determination of urine urate concentrations D. Radiologic examination of the affected joint E. Response to IV corticosteroids

The correct answer is A. The first acute attacks of gout are typically seen on the first metatarsophalangeal joint (an ailment known as podagra). Patients tend to be middle-aged men. Some risk factors include increased alcohol consumption and use of thiazide diuretics. In the acute phase of gout, as seen in this patient, aspiration of the joint will show negatively birefringent crystals in plane-polarized light within neutrophils in the aspirated synovial fluid. Clinical Pearls Gout (monosodium urate crystal deposition disease) is characterized biochemically by extracellular fluid urate saturation. Aspiration of the affected joint revealing negatively birefringent crystals of monosodium urate is the definitive test identifying gout as the cause of the patient's symptom. The clinical manifestations include one or more of the following: All patients with gout have hyperuricemia (saturation of serum for urate) at some point in their disease. Hyperuricemia can be caused by impaired renal excretion or overproduction of uric acid and/or overconsumption of purine-rich foods that are metabolized to urate. In some hyperuricemic individuals, a period of hyperuricemia leads to monosodium urate (MSU) crystal deposition, reaction to which can result in the acute and/or chronic inflammation associated with the signs and symptoms of gout. In some patients, benefit may be derived from avoidance of specific dietary components that raise serum urate levels, such as organ-rich foods (e.g., liver, sweetbreads), beer, or distilled spirits.

An older athlete will not be playing in his weekly basketball game because he has a ruptured disc. Which of the following correctly describes the gross pathology of this condition? A. The anulus fibrosus protrudes anteriorly through a weakened nucleus pulposus B. The anulus fibrosus protrudes anteromedially through a weakened nucleus pulposus C. The anulus fibrosus protrudes posteriorly through a weakened nucleus pulposus D. The nucleus pulposus protrudes anteriorly through a weakened anulus fibrosus E. The nucleus pulposus protrudes posterolaterally through a weakened anulus fibrosus

The correct answer is E. Herniated intervertebral discs, often called slipped or ruptured discs by lay people, occur when the weakened outer anulus fibrosus of the disc permits herniation of the centrally located nucleus pulposus. The defect typically occurs in a posterolateral direction, where it may compress an adjacent spinal nerve root, causing severe low back or leg pain.

A 47-year-old man presents with acute pain in his big toe. Laboratory tests reveal a uric acid level of 10 mg/dL. He is started on a medication. Several days after the resolution of the acute attack, he is started on a second medication that decreases uric acid in both the serum and the urine. Which of the following is the second medication? A. Allopurinol B. Colchicine C. Indomethacin D. Probenecid E. Sulfinpyrazone

The correct answer is A. This is a simple drug mechanism question made somewhat more difficult by the additional information presented in the stem. First, the diagnosis of gout should have been easily made. (Other classic clues for gout include negatively birefringent needle-shaped crystals and the presence of tophi.) The only crucial information in the question is that Drug 2 decreases uric acid in both the serum and the urine. Allopurinol inhibits the formation of uric acid by blocking the conversion of hypoxanthine to xanthine and xanthine to uric acid by xanthine oxidase (allopurinol is an analog of hypoxanthine). If uric acid synthesis is decreased, you would expect to see a decrease in both the serum and urine levels. Note that allopurinol is used in the treatment of severe hyperuricemia, whether or not it is related to gout. Also note that in acute attacks of gout, therapy with allopurinol should be delayed until after the acute episode has resolved and the patient is on maintenance doses of colchicine (probably Drug 1 in this vignette).

A 42-year-old patient has had recurrent nocturnal attacks of acute arthritis involving the right first metatarsophalangeal joint, with serum uric acid levels >7.5 mg/dL. He develops slowly growing subcutaneous nodules in his feet and olecranon. Aspiration of a nodule reveals crystals of sodium urate. Which of the following drugs will most likely shrink, and possibly eliminate, these nodules? A. Allopurinol B. Aspirin C. Colchicine D. Indomethacin E. Thiazide diuretics

The correct answer is A. This patient has gouty arthritis, which most frequently affects the first metatarsophalangeal joint and manifests with acute pain and swelling, often at night. Elevated serum urate levels and the presence of urate crystals within subcutaneous nodules are confirmatory. These subcutaneous nodules, called tophi, regress with appropriate treatment aimed at decreasing urate levels. Allopurinol inhibits xanthine oxidase, thus preventing formation of uric acid.

A 4-year-old girl is brought to the office because of new-onset pain and swelling in her left knee for 1 week. She has been complaining of knee pain every morning for the past 7 days. It is worst when she wakes up and she is reluctant to get up from bed to walk. Within an hour, the pain improves and she is fine for the rest of the day. Her parents had tried giving her acetaminophen but it did not help. She had just recovered from an upper respiratory tract illness and was doing well except for being a little tired and sleeping longer. The parents deny any trauma or injury to her knee. She has no significant past medical history and takes no medication. On physical examination the patient appears in no distress. Her vital signs are within normal limits and her blood pressure is 90/60 mm Hg. Physical examination reveals a mildly edematous and warm left knee with decreased ROM. Routine blood work yields an erythrocyte sedimentation rate of 57 mm/h in the first hour. A complete blood count and urine analysis are unremarkable. Radiography of the knee does not show any destructive changes. Ophthalmologic examination shows bilateral asymptomatic nongranulomatous uveitis.Which of the following findings would indicate a favorable prognosis in this patient? A. Antinuclear antibody positive B. Fever C. Hepatosplenomegaly D. HLA-B27 positive E. Rheumatoid factor positive

The correct answer is A. This patient has oligoarticular juvenile rheumatoid arthritis, which has an excellent prognosis when associated with positive antinuclear antibodies (ANA). Juvenile rheumatoid arthritis is a chronic nonsuppurative inflammation of the synovium of the joints. It is characterized by joint effusions, destruction of joint cartilage, and bone deformity, destruction, and fusion. Although a specific etiology is not known, a pre-existing susceptibility followed by an environmental trigger is believed to be the trigger. Infectious triggers include viruses, Borrelia, and Mycoplasma. An autoimmune origin is also suggested. Initial presentation includes morning stiffness, joint pain, and swelling. The joint is warm and has decreased ROM. Erythema is rare. Low-grade fever may be present, together with malaise. Patients who have positive ANA are more susceptible to uveitis. Clinical Pearls Juvenile idiopathic arthritis is the most common chronic arthropathy of children, affecting 1 in 1,000 children. Oligoarticular disease appears to have the best prognosis with better functional outcome and a higher percentage of patients who achieve remission. Uveitis is the most common extra-articular manifestation of juvenile arthritis; the presence of antinuclear antibodies implies an excellent prognosis. In oligoarticular juvenile rheumatoid arthritis, less than four joints are affected. Onset of the disease is either insidious or abrupt, and presentation includes morning stiffness, joint pain, and lethargy. On physical examination joint swelling and decreased ROM of the affected joint are appreciated.

A 38-year-old woman complains of cold and painful fingertips, as well as difficulty swallowing and indigestion. Physical examination is remarkable for a thickened, shiny epidermis over the entire body, with restricted movement of the extremities, particularly the fingers, which appear claw-like. Which of the following autoantibodies will likely be found in this patient's serum? A. Anti-DNA topoisomerase I (anti-Scl-70) B. Anti-double-stranded DNA (ds DNA) C. Anti-IgG D. Anti-Sm E. Anti-SS-A

The correct answer is A. This patient has systemic sclerosis, also called scleroderma. Antibodies to topoisomerase I (anti-Scl-70) occur in up to 70% of patients who have diffuse systemic sclerosis, but only rarely in other disorders. Systemic sclerosis is characterized initially by excessive fibrosis and edema of the skin, especially the hands and fingers, producing sclerodactyly (characteristic changes in the fingers, which resemble claws). Raynaud phenomenon is common. The diffuse type of systemic sclerosis generally spreads to include visceral organs, such as the esophagus (producing dysphagia), the lungs (producing pulmonary fibrosis), the heart (leading to heart failure or arrhythmia), and the kidneys (renal failure causes 50% of scleroderma deaths). Women are affected more than men (3:1 ratio). A more restricted variant of systemic sclerosis with a somewhat more benign course is CREST syndrome (Calcinosis, Raynaud syndrome, Esophageal dysmotility, Sclerodactyly, and Telangiectasia), characterized by the presence of anti-centromere antibodies (although 10% of CREST patients will have anti-topoisomerase antibody as well). Clinical Pearls Scleroderma or systemic sclerosis is a chronic, multisystem disease. It begins nonspecifically with fatigue, vague musculoskeletal complaints, and diffuse swelling of the hands and Raynaud phenomenon. Risk factors include family history and presence of immune dysregulation (+ANA). Physical examination signs include skin thickening, painless loss of hand function, sclerodactyly, heartburn, reflux, and dysphagia, arthralgias and myalgias, and subcutaneous calcinosis. Diagnostic tests include ANA, anti-Scl 70 or anti-topoisomerase I, and anti-RNA polymerase 3 antibody testing.

A 38-year-old man with a history of genital herpes comes to the health care provider because of right knee pain and swelling. He reports that the symptoms started earlier in the day and it is extremely painful to move or even touch his knee. He denies any trauma to the affected knee. He also reports recent unprotected sex with a prostitute. His temperature is 39°C (102.2°F), blood pressure 125/90 mm Hg, pulse 108/min and regular, and respirations 15/min. Physical examination reveals a swollen and tender right knee with a palpable effusion. He is tachycardic but his examination is otherwise normal. An arthrocentesis is performed and fluid is sent for analysis. Which of the following laboratory findings is most likely to be found for this patient? A. Cell count with 75,000 WBCs B. Culture positive for gram-positive cocci in clusters C. Gram stain revealing gram-negative diplococci D. Negative birefringent crystals E. Rhomboid-shaped crystals

The correct answer is A. This patient likely has a septic arthritis. He is complaining of monoarticular swelling and pain. He also has a high fever that suggests the diagnosis. Most patients age <40 who have septic arthritis have gonococcal disease, especially if there is a history of unprotected sex. The diagnosis of septic arthritis is made by finding a WBC count >50,000. In other cases of septic arthritis, staph and strep are the most likely pathogens identified. Clinical Pearls Patients who have a painful, hot, swollen, and restricted motion joint should have aspiration of the joint. Septic joint can occur from direct inoculation or via hematogenous spread. The most common causative organism responsible for septic arthritis in the sexually active age group is Neisseria gonorrhoeae. The arthrocentesis will show a significantly increased WBC count. Patients who have septic arthritis without an increased WBC count are not ruled out for septic arthritis in this setting if septic arthritis is under serious consideration. When in doubt, the red hot joint should be treated as a septic joint.

A 40-year-old woman has an 8-month history of diffuse joint pain and swelling that involves both hands and knees. She states that she cannot get any work done in the morning because of the pain, but the pain usually subsides as the day progresses. She tires easily and constantly feels "feverish." Her temperature is 37.8ºC (100ºF), blood pressure 110/70 mm Hg, pulse 60/min, and respirations 18/min. Physical examination shows tender, swollen, and "boggy" hands and knees. Laboratory studies show hematocrit 34% (normal 36-44%) and hemoglobin 10 g/dL (normal 12.1-15.1). Radiograph of the knee is most likely to show which of the following? A. Bone erosions B. Osteophyte formation C. Subchondral cyst formation D. Subchondral sclerosis E. Subchondral tophi

The correct answer is A. This patient most likely has rheumatoid arthritis, a chronic inflammatory disorder characterized by symmetric joint involvement and extra-articular manifestations. It most commonly affects middle-aged women. Symptoms include joint pain and swelling, low-grade fever, malaise, fatigue, vasculitis, pericarditis, rheumatic nodules, episcleritis, and scleritis. Laboratory findings include normochromic, normocytic anemia, increased erythrocyte sedimentation rate, and serum rheumatoid factors. The proximal interphalangeal joints (PIP), metacarpophalangeal joints (MCP), and the wrist joints are the most commonly involved joints. Distal interphalangeal joints (DIP) are usually spared. Clinical Pearls Rheumatoid arthritis (RA) is an autoimmune disease that typically presents as joint pain in the morning with symptoms lasting more than 30 minutes. In osteoarthritis (OA), joint pain improves in <30 minutes (Gel sign). RA is usually accompanied by constitutional findings and other autoimmune symptoms, whereas OA is not. Bony erosions and periarticular osteopenia seen on radiograph are characteristic of RA. Joints in rheumatoid arthritis are typically symmetrically involved. Joints affected by osteoarthritis can have osteophyte formation, which form as the body tried to protect the joints from ongoing bone on bone destruction.

A 27-year-old woman is admitted to the hospital because of 4 days of joint pain. Initially she had diffuse joint pain that would "travel" from her wrists to her elbows and to her knees. Over the last 24 hours, however, she has had intense pain and swelling localized to her right knee. She denies any trauma to the area. She denies any fevers or chills but admits to mild dysuria that she attributed to a urinary tract infection. Further history reveals that she has had unprotected sex with 4 partners in the last 3 months. Her temperature is 37.0ºC (98.6ºF), blood pressure 120/70 mm Hg, pulse 113/min, and respirations 18/min. The right knee is swollen and tender. She has scattered necrotic pustules on her palms and soles. Arthrocentesis is performed and a complete blood count and blood cultures are obtained. Hgb 14 WBC 10,000 Platelets 170,000 Hct 36% Gram stain of the synovial fluid is shown. Which of the following is the most appropriate next step in management? A. Begin therapy with IV ceftriaxone (Rocephin) B. Begin therapy with IV nafcillin C. Begin therapy with IV penicillin D. Begin therapy with IV vancomycin E. Hold antibiotics and wait for culture and sensitivities results

The correct answer is A. This patient's presentation is classic for disseminated gonococcal disease. Disseminated gonococcal disease begins as a migratory arthritis and evolves into a purulent monoarthritis. Fever, elevated WBCs, and genitourinary symptoms are variable. Necrotic papules on the extremities can be a valuable clue to the diagnosis. The patient's history of unprotected sex and dysuria should provide further clues to diagnosis. NAAT (nucleic acid amplification test) is recommended on the first urine in the morning in order to make this diagnosis. Health care providers should culture the rectum, cervix, urethra, and pharynx as well as obtain blood cultures if a drug-resistant strain of gonorrhea is suspected. Initial treatment with ceftriaxone is now recommended as first-line treatment. Typically, this is accompanied with a single one gram of azithromycin. Parenteral antibiotics are typically given for 7 days. It would also be prudent to cover for the possible coexisting Chlamydia infection with the single dose of azithromycin. Clinical Pearls Gonorrhea can lead to systemic symptoms such as polyarthritis, which indicates disseminated disease. The joints most commonly involved include the wrists, ankles, and small joints of the hands and feet. Signs of disseminated disease can also include conjunctivitis, fever, and skin lesions at the extremities. Treatment includes ceftriaxone (Rocephin) plus a single one-gram dose of azithromycin. Disseminated gonococcal infection (DGI) is the most common cause of acute nontraumatic monoarthritis or oligoarthritis in young adults. Patients who have DGI typically present with one of 2 syndromes: A triad of tenosynovitis, vesiculopustular skin lesions, and polyarthralgias without purulent arthritis Purulent arthritis without associated skin lesions Gonococcal arthritis is a common cause of septic arthritis in which the organism cannot be cultured on routine culture media. Cultures of synovial fluid tend to be positive in <50% of cases of gonococcal arthritis. Because of this, a negative synovial culture does not rule out gonococcal arthritis. Affected adults are sexually active individuals and typically present with fever, chills, skin lesions, polyarthralgias, and tenosynovitis evolving into a persistent monoarthritis or oligoarthritis. The joint aspirate should be cultured for N. gonorrhoeae when the history is suggestive. NAAT of the first morning voided urine can prove that gonorrhea is present. The yield can be increased if plates of chocolate agar or Thayer-Martin medium are inoculated with synovial fluid at the bedside along with cultures from clinically appropriate sites (e.g., the pharynx, urethra, cervix, rectum, and skin lesions). Blood cultures are often positive in patients who have DGI presenting with tenosynovitis and skin lesions alone, but are frequently negative if a joint effusion is present. The infected fluid is usually purulent with an average leukocyte count (most of which are neutrophils) of 50,000 -150,000 cells/mm3. The likelihood of septic arthritis increases with rising synovial fluid leukocyte count. Nucleic acid hybridization tests and nucleic acid amplification tests (NAAT) are now generally recommended as the first-line method of testing. Urine, urethral, cervical, and vaginal specimens can be utilized.

A term newborn is delivered vaginally following a breech presentation. The pediatric health care provider is called to the newborn nursery because on physical examination, the Barlow test is positive for bilateral subluxation of the hips. There is decreased abduction of both hips. Besides breech presentation, which of the following infants are most at risk for this condition? A. African American infants B. Female infants C. Infants of mothers who have preeclampsia D. Premature infants E. Second-born infants

The correct answer is B. Developmental dysplasia of the hip generally includes subluxation (partial dislocation) of the femoral head, acetabular dysplasia, and complete dislocation of the femoral head from the acetabulum. The incidence of developmental dysplasia of the hip is about 1 in 1,000 live births. The disorder is more common in females than in males, in many series, as much as 5 times more common. The combination of female sex and breech presentation results in developmental dysplasia of the hip in 1 in 35 such births. Developmental dysplasia of the hip is also more common in firstborn children than in subsequent siblings (compare with choice E). A family history increases the likelihood of this condition to approximately 10%. In newborns (<6 months), it is important to perform a careful clinical examination, because x-rays are not absolutely reliable in diagnosing congenital dysplasia of the hip in this age group. Routine clinical screening should include both the Ortolani test and the provocative maneuver of Barlow. The Ortolani test that is performed by gently abducting and adducting the flexed hip detects any reduction or dislocation of the femoral head from the true acetabulum. The provocative maneuver of Barlow detects any potential subluxation or posterior dislocation of the femoral head by direct pressure on the longitudinal axis of the femur while the hip is in adduction. Clinical Pearls Developmental dysplasia of the hip (DDH) describes a spectrum of conditions in which the femoral head has an abnormal relationship to the acetabulum, resulting in hip instability. Risk factors for DDH include: Female sex, with women outnumbering men by a 4:1 margin Breech presentation caused by mechanical factors Family history of DDH due to genetic factors Limited fetal mobility for example, breech, and also increased incidence in first-born infants (less space in primigravida uterus)

A 70-year-old woman undergoes a bilateral hip replacement for osteoarthritis. Which of the following pathologic changes will the removed femoral heads most likely demonstrate? A. Marked synovial proliferation with pannus formation B. Multiple small fractures in the cartilage C. Multiple white flecks in the synovium D. No visible change E. Pus covering the articular surface

The correct answer is B. Osteoarthritis is characterized by mechanical, rather than inflammatory, damage to the joint. The damage usually begins as multiple small fractures of the cartilage tips of the involved bones, which can lead to wearing down of the cartilage to expose the underlying bone. In longstanding cases, the articular ends of the bones may develop a mushroom-like or flattened deformation.

Patients who have the HLA-B27 antigen are known to have an increased risk for developing certain pathologic conditions. Which of the following infectious agents may cause a syndrome characterized by arthritis, conjunctivitis, and urethritis in an HLA-B27-positive patient? A. Borrelia burgdorferi B. Chlamydia trachomatis C. Enterotoxigenic Escherichia coli D. Group A beta-hemolytic streptococci E. Trichomonas vaginalis

The correct answer is B. Reactive arthritis is a serious sequelae that may follow enteric infections caused by Shigella, Salmonella, Yersinia, or Campylobacter or sexually transmitted diseases caused by Chlamydia or Ureaplasma. Up to 80% of cases occur in HLA-B27-positive individuals. Male predominance is characteristic in cases following sexually transmitted infections. The role of other types of infection is not clear, and infectious organisms have not been cultured from affected joints. Clinical Pearls Patients who have reactive arthritis develop an inflammatory condition after exposure to certain GI and genitourinary infections including Chlamydia species, Campylobacter species, Salmonella, Shigella, and Yersinia. Arthritis that occurs is usually asymmetric, oligoarticular, and mostly affects the larger joints of the lower extremity. Patients may also develop enthesitis, an inflammation at sites where tendons insert into bones.

During a football game, a player sustains a powerful blow to the lateral side of his weight-bearing leg. He experiences excruciating knee pain and is unable to walk. The 3 structures most likely to be injured are which of the following? A. Anterior cruciate and lateral collateral ligaments and the lateral meniscus B. Anterior cruciate and medial collateral ligaments and the medial meniscus C. Posterior cruciate and lateral collateral ligaments and the lateral meniscus D. Posterior cruciate and medial collateral ligaments and the lateral meniscus E. Posterior cruciate and medial collateral ligaments and medial meniscus

The correct answer is B. The anterior cruciate and medial collateral ligaments and the medial meniscus are sometimes called the unhappy triad because they are commonly injured by lateral blows to the knee that forcefully abduct the tibia. An unhappy triad (also called terrible triad, O'Donoghue's triad, or "blown knee") is an injury to the knee. It commonly occurs in contact sports. The mechanism for this injury occurs when a lateral (outside) force to the knee is received while the foot is fixed on the ground in external rotation. The classic O'Donoghue's triad is characterized by an injury to three knee structures: the anterior cruciate ligament the medial collateral ligament the medial meniscus

A 36-year-old man develops rapid mental status deterioration 2 days after sustaining a femoral fracture in a skiing accident. Physical examination shows multiple petechiae in the anterior chest and abdomen. On the third day, the patient lapses into coma and dies. Postmortem examination of the brain reveals numerous petechial hemorrhages in the corpus callosum and centrum semiovale. Which of the following is the most likely diagnosis? A. Diffuse axonal injury B. Fat embolism C. Septic embolism D. Systemic thromboembolism E. Watershed infarction

The correct answer is B. The clinical manifestations in the scenario are consistent with fat embolism. This complication is frequently associated with fractures of long bones, but this is usually asymptomatic. Fat embolism mainly affects the lungs and the brain, and the clinical picture consists of dyspnea, tachycardia, and mental status changes. Only rarely does this condition lead to death. In the lungs, fat emboli can be visualized histologically. In the brain, multifocal petechiae in the white matter represent the most common pathologic change. Clinical Pearls Fat embolism is a clinical diagnosis. It usually presents with a history of trauma to long bones or pelvis (including orthopedic procedures). Symptomatic patients present with respiratory distress, neurologic impairment, and a petechial rash over the upper body. Petechial rash, present in only 20% of cases, is considered pathognomonic in the setting of long-bone trauma.

A 24-year-old man presents with pain in his right wrist that resulted when he fell hard on his outstretched hand. Radiographic studies indicate an anterior dislocation of a proximal row carpal bone that articulates with the most lateral proximal row carpal bone. Which of the following bones was dislocated? A. Capitate B. Lunate C. Scaphoid D. Trapezoid E. Triquetrum

The correct answer is B. The lunate is in the proximal row and articulates with the scaphoid laterally (this being the most lateral of the proximal row). The lunate is the most commonly dislocated carpal bone. It is usually displaced anteriorly by rotation on its proximal, convex surface (where it articulates with the radius). The displaced bone may compress the median nerve in the carpal tunnel, leading to pain, sensory loss, and/or paralysis. Clinical Pearls The proximal carpal bones (from radial to ulnar) are: Scaphoid Lunate Triquetrum (dorsal) Pisiform (volar) The distal carpal bones (from radial to ulnar) are: Trapezium Trapezoid Capitate Hamate

An elderly man is involved in a rear-end automobile collision in which he hyperextended his neck. He develops paralysis and burning pain of both upper extremities, but maintains good motor function in his legs. Which of the following is the most likely diagnosis? A. Anterior cord syndrome B. Central cord syndrome C. Posterior cord syndrome D. Reflex sympathetic dystrophy E. Spinal cord hemisection

The correct answer is B. The mechanism of injury (hyperextension) and the relative sparing of the lower extremities in the presence of upper extremity deficits are classic for central cord syndrome. Clinical Pearls The central cord syndrome is characterized by loss of pain and temperature sensation in the distribution of one or several adjacent dermatomes at the site of the spinal cord lesion caused by the disruption of crossing spinothalamic fibers in the ventral commissure. Dermatomes above and below the level of the lesion have normal pain and temperature sensation, creating the so-called suspended sensory level. Vibration and proprioception are often spared. Central cord syndrome classically causes more motor impairment in the upper as opposed to lower extremities. There is variable degrees of sensory losses for dermatomes located below the area of injury. Injuries of this type can occur with even mild injury to the neck for patients who have underlying cervical spondylosis.

During an automobile accident, a person sustains a cervical spine fracture and dies after a small bony fragment is driven into his spinal cord. From which of the following bones was this fragment most likely derived? A. Atlas B. Axis C. Seventh cervical vertebrae D. Sixth cervical vertebrae E. Third cervical vertebrae

The correct answer is B. The odontoid process is the part of the axis (second cervical vertebra) that fits into and articulates with the atlas (first cervical vertebra). It is susceptible to traumatic fracture, and bony fragments can injure the spinal cord. The scenario presented is unfortunately realistic, and a similar mechanism is thought to cause deaths by execution-style hanging. Clinical Pearls Traumatic spondylolisthesis of the axis or C2 (so-called "hangman's fracture") is an unstable injury that occurs when the head and proximal neck is thrown into extreme hyperextension as a result of abrupt deceleration. The mechanism of a hangman's fracture is a combination of hyperextension and distraction. Traumatic spondylolisthesis is a common C-spine injury in fatal MVAs. There may be additional C-spine pathology. Craniofacial and vertebral artery injuries are not uncommon findings.

Over a span of years, a woman develops thickening of the fascia of one palm. It eventually causes a flexion contracture of the hand, most markedly involving the fourth and fifth fingers. The patient most probably has which of the following conditions? A. Desmoid B. Dupuytren's contracture C. Neurofibroma D. Peyronie disease E. Plantar fibromatosis

The correct answer is B. The patient has palmar fibromatosis, also known as Dupuytren's contracture. This is a benign condition that may stabilize spontaneously or may require surgical excision. Incidence is highest in patients age >40 who have chronic diseases (including diabetes and liver disease).

An 80-year-old woman experiences the sudden onset of lower extremity weakness and collapse on getting out of bed. She has been previously healthy and has not taken any medications. She has no history of back pain. On examination she is alert and oriented to time, place, and date. Her upper extremity sensation and strength are intact. Her legs are weak bilaterally with loss of pain and temperature sensation and she has areflexia. Her bladder is distended. Which of the following is the most likely diagnosis? A. Anterior cerebral artery occlusion B. Anterior spinal artery occlusion C. Cauda equina syndrome D. Guillain-Barré syndrome E. Thoracic spinal cord compression

The correct answer is B. The perfusion territory of the anterior spinal artery includes the anterior horn cells and part of the pain and temperature pathways. Thrombosis of this artery causes flaccid paralysis, loss of bowel and bladder function, and loss of pain and temperature sensation. Clinical Pearls The occlusion of the anterior spinal artery may present with flaccid paralysis, loss of bowel and bladder function, and loss of pain and temperature sensation, as the supplying artery supplies the anterior horn cells, which control part of the pain and temperature pathways.

A 26-year-old man is stabbed in the left chest during a bar brawl. Several days after he is treated, he returns to the health care provider because of decreased function in his left arm. Physical examination reveals a winged left scapula and an inability to raise his left arm above the horizontal. Which of the following nerves is most likely affected? A. Axillary B. Long thoracic C. Lower subscapular D. Suprascapular E. Thoracodorsal

The correct answer is B. The serratus anterior, innervated by the long thoracic nerve, is responsible for stabilization of the scapula during abduction of the arm from 90 to 180 degrees. When the long thoracic nerve is damaged, it is difficult to elevate the arm above the horizontal. This nerve arises from C5, C6, and C7. Remember: a winged scapula is a classic clue for long thoracic nerve injury. Clinical Pearls The C5 through T1 roots emerge from the spinal column and give off proximal, small branches that contribute to form the long thoracic nerve, which innervates the serratus anterior muscle, inserts on the vertebral border of scapula, and functions to draw the scapula forward. Long thoracic neuropathy usually produces winging of the scapula, which results from an inability of the serratus anterior to draw the scapula forward. Patients with this injury also have problems raising their upper extremity above the horizon.

A 40-year-old left-handed dominant woman comes to the office with complaints of burning and tingling sensations in the left hand for several months. She relates that she has been frequently awakened at night by aching pain in the same hand. She is otherwise in good health. Examination fails to detect any impairment in sensation, but pain is elicited by extreme flexion of the wrist. The patient is unable to correctly identify different fabrics by rubbing between the left thumb and index finger. She has decreased ability to hold opposition strength when her thumb and fifth digit are brought together. Which of the following is the most likely diagnosis? A. Angina pectoris B. Carpal tunnel syndrome C. Dupuytren's contracture D. Fibrositis E. Reflex sympathetic dystrophy

The correct answer is B. The symptomatology is classic for carpal tunnel syndrome, which is a form of neuropathy resulting from anatomic compression of the median nerve. Pain, tingling sensations, and hypoesthesia in the distribution of the median nerve are the cardinal manifestations. These often undergo exacerbations at nighttime. A shock-like pain upon percussion on the volar aspect of the wrist (Tinel sign) is an additional characteristic sign. Phalen sign reveals increased pain in the median nerve distribution when the wrist is forcefully flexed. Carpal tunnel syndrome is most often idiopathic, but may represent a manifestation of underlying disorders such as rheumatoid arthritis, sarcoidosis, amyloidosis, acromegaly, and leukemia. Clinical Pearls Carpal tunnel syndrome (CTS) refers to a complex of signs and symptoms brought on by compression of the median nerve as it travels through the carpal tunnel. Patients commonly experience pain and paresthesia, and less commonly weakness, in the median nerve distribution. CTS involves the first three digits and the radial half of the fourth digit. The symptoms are typically worse at night and characteristically awaken affected patients from sleep. CTS symptoms are often provoked by activities that involve flexing or extending the wrist or raising the arms. Clinical signs may include weakness of thumb abduction and opposition, weakness of flexing the index and middle fingers at the MCP joints, and atrophy of the thenar eminence. Provocative maneuvers for CTS include the Phalen, Tinel, manual carpal compression, and hand elevation tests. Initial conservative treatment includes nocturnal wrist splinting in the neutral position. Surgery can be done via median nerve release for patients who do not respond to conservative care. Patients who fail conservative therapy who wish to have carpal tunnel release need to have nerve compression testing or electromyographs performed in order to document damage to the median nerve before surgery can be approved. Patients who sustain a Colles' fracture, which involves dorsal displacement of the distal radius segment who undergo surgery typically have median nerve release at the time of surgery to prevent carpal tunnel syndrome from occurring.

A construction worker has a serious spinal injury in which his rib cage and abdominal muscles become completely paralyzed. He is still able to breathe, however, because his diaphragm continues to contract. At which level might his spinal cord injury have occurred? A. C2 B. C7 C. L3 D. T5 E. T12

The correct answer is B. Trauma to the lower cervical cord (at C7, for example) can cause the pattern described in the question stem, because the lesion is below the origin of the phrenic nerve, but above the origin of the nerves innervating the muscles of the rib cage and abdomen. Trauma high in the neck at C2 (choice A), above the origin of the phrenic nerve (C3-C5 nerve roots), would cause diaphragmatic paralysis as well. Clinical Pearls The major muscle that mediates inspiration is the diaphragm, which is innervated by the phrenic nerve (third to fifth cervical roots). Patients with proximal spinal cord injury can sometimes utilize oral, pharyngeal, and laryngeal muscles for short-term ventilation by projecting boluses of air past the glottis (glossopharyngeal breathing), although these muscles are not traditionally thought of as muscles of respiration. Due to complete paralysis of the respiratory muscles, patients with SCI above the C3 root have acute ventilatory failure and do not survive unless manual ventilation is rapidly instituted.

A 24-year-old woman is seen in the hospital for a broken femur. The patient was in a motor vehicle accident 20 hours ago and was brought to the hospital by EMS. On the scene, she was found belted in her car in the driver's seat, and her only documented injury was the fracture in the thigh. She had no loss of consciousness or altered mental status. On arrival to the hospital, radiographs confirmed a fracture of her femur. She was stabilized overnight and scheduled for surgery the next day. Which of the following is the major surgical risk for this patient? A. Air embolism B. Cerebrovascular accident C. Fat embolism D. Osteomyelitis E. Permanent disability Show/hide explanation

The correct answer is C. If a transesophageal echo probe is placed in every patient undergoing femoral reaming for fracture repair, the incidence of fat and particle debris in the right atrium approaches 70%. In fact, a major risk from lower extremity orthopedic procedures is pulmonary embolism caused by fat or clots. The intramedullary pressures generated during the repair are >500 psi and are enough to cause venous extrusion of fat (fat embolism) and other particulate matter into the circulation. Clinical Pearls Fat embolism syndrome (FES) is most commonly associated with long bone and pelvic fractures and is more frequently seen in closed fractures than open fractures. Fat emboli may be the result of fat globules entering the bloodstream through tissue that has been disrupted by trauma or alternatively via production of the toxic intermediaries of plasma-derived fat. The classic triad of FES includes hypoxemia, neurologic abnormalities, and a petechial rash. FES is a clinical diagnosis that can be made when the pathognomonic petechial rash occurs in an appropriate clinical setting. There is no diagnostic test that is sufficiently sensitive or specific to be useful for confirming or excluding FES. Appropriate preventive management strategies against FES include corticosteroids, early operative intervention, early mobilization following surgery, and limitation of the intraosseous pressure during orthopedic procedures. Supportive care alone is the mainstay of therapy for FES. Most patients with FES fully recover.

An 80-year-old woman complains of a 4-month history of worsening gait and low back pain that is worse on walking. She denies any trauma and is not incontinent. She has been fairly healthy and only takes iron supplements. On examination, she has hypoactive muscle stretch reflexes in the legs. The plain radiographs of the lumbosacral region show degenerative changes that appear to be age-appropriate. Which of the following is the most likely diagnosis? A. Acute lumbar disc herniation B. Cervical stenosis C. Lumbar stenosis D. Myopathy E. Normal pressure hydrocephalus (NPH)

The correct answer is C. Lumbar stenosis is caused by degenerative changes in the lumbosacral spine. The history is that of vague low back pain with subtle physical examination findings referable to impingement on motor and sensory roots. Neurogenic claudication occurs with worsening of symptoms with walking and relief of symptoms with rest and leaning forward. Clinical Pearls Spinal stenosis occurs as a result of degenerative changes in the lumbar spine. Neurogenic claudication occurs with low back pain and leg pain with lower extremity paresthesias worsened by walking and relieved by rest. MRI can demonstrate neural compression in the spinal canal and foramina. Initial therapy can be conservative with NSAIDs followed by epidural injection or decompressive spinal surgery for treatment failure.

A 72-year-old man of Norwegian ancestry has a contracted hand that can no longer be extended and placed flat on a table. The problem developed gradually, over many years. He complains of no pain or neurologic abnormalities and, to the extent that the deformity allows, can move his fingers at will. Physical examination demonstrates the deformity described and in addition shows the presence of palpable fascial nodules. Which of the following is the most likely diagnosis? A. Carpal tunnel syndrome B. De Quervain tenosynovitis C. Dupuytren's contracture D. Palmar tenosynovitis E. Rheumatoid arthritis

The correct answer is C. Older men of Scandinavian descent are the victims of this otherwise mysterious contracture. The palpable nodules are classic, as is the deformity itself. There is an increase in this condition in patients who have a northern European heritage. Clinical Pearls Dupuytren's contracture is a relatively common, benign, slowly progressive fibroproliferative disease of the palmar fascia. Initial tendon thickening is painless and often goes unnoticed and undiagnosed. As the scarring process progresses, nodules form on the palmar fascia and the finger gradually loses its flexibility with contractures that draw one or more fingers into flexion at the metacarpophalangeal (MP) joint. The cause of Dupuytren's contracture is unknown. It exhibits a pronounced genetic predisposition and is more common in northern Europeans.

A 15-year-old girl is brought to the health care provider with a 2-day history of pain and swelling in her left knee. She plays soccer regularly on her school team. There is no history of trauma. On physical examination there is marked swelling and tenderness over the anterior tibial tuberosity. Radiograph of her left knee reveals irregularities of the tubercle contour and haziness of the adjacent metaphyseal border. Which of the following is the most likely explanation for these symptoms? A. Avascular necrosis of the hip B. Legg-Calve-Perthes disease C. Osgood-Schlatter disease D. Septic arthritis E. Slipped capital femoral epiphysis

The correct answer is C. Osgood-Schlatter disease is a common cause of anterior knee pain in the adolescent. It is a stress reaction of the insertion of the patella tendon into the tibia tubercle. The condition previously was more common in adolescent boys near the time of their growth spurt; however, girls are being affected more frequently because of their increased involvement in sports. Often there is only a tendonitis of the patella tendon insertion and inflammation of the deep infrapatellar bursa. Clinically, a young teenager who is quite active in sports will complain of pain and swelling over the tibial tubercle. The condition may be unilateral or bilateral. On examination there is an obviously swollen, tender, and occasionally warm area over the tibia tubercle. Radiograph can be normal or show fragmentation of the area of patella tendon insertion. The natural history of untreated Osgood-Schlatter is relatively benign, if allowed to heal and complications are avoided. Because unusual stress at a time of rapid growth is occurring, limiting the offending sports activity is usually sufficient to relieve the symptoms. Kneeling and squatting should be limited. Physical therapy is occasionally used if there is unusual hamstring tightness. Clinical Pearls Osgood-Schlatter disease is an overuse injury caused by repetitive strain and chronic avulsion of the secondary ossification center of the tibial tubercle. Osgood-Schlatter occurs most frequently in sports that involve running, cutting, and jumping and in those who have recently undergone a rapid growth spurt. The most common presenting complaint is anterior knee pain that increases gradually over time; the pain is exacerbated by direct trauma, kneeling, running, jumping, squatting, climbing stairs, or walking uphill and is relieved by rest. The diagnosis of Osgood-Schlatter is made clinically. The characteristic findings are tenderness and soft tissue or bony prominence of the tibial tubercle in a patient with an otherwise normal examination. Imaging is typically not necessary. Symptoms generally resolve once the growth plate is ossified. Conservative measures are the mainstay of therapy including ice, rest, anti-inflammatories, and possibly physical therapy to tighten the supporting muscles around the insertion site.

A 60-year-old woman complains of increasing weakness and fatigue over the past several months. She reports that any physical activity leads to fatigue, and she is unable to find any comfortable position at rest. She mostly complains of neck, shoulder, and hip stiffness and pain. She admits having difficulty getting out of a chair. She has lost 5 pounds over the past 3 months. Prior to this, she has always been healthy and is not on any medications. Physical examination is unremarkable. Which of the following is the most likely diagnosis? A. Depression B. Hypokalemia C. Polymyalgia rheumatica D. Polymyositis E. Temporal arteritis

The correct answer is C. Polymyalgia rheumatica (PMR) is characterized by muscle discomfort, and patients often have vague symptoms. The neuromuscular examination is normal. Patients will often have complaints of weakness but subjective testing of these patients does not demonstrate actual weakness. The muscles are not tender on palpation. Clinical Pearls Polymyalgia rheumatica is an inflammatory rheumatologic syndrome manifested by pain and morning stiffness primarily in the neck, shoulder, and hip areas. This condition is rarely seen age <50 and predominates in women. Although pain and stiffness predominate, there is no measureable weakness on physical examination testing. SED rate is elevated but not muscle enzymes. Steroids dramatically improve symptoms. Giant cell arteritis is a known complication.

A patient has severe arthritis involving the lower back. Before making a diagnosis of ankylosing spondylitis, the patient should be questioned about which of the following diseases? A. Carcinoid syndrome B. Gallbladder disease C. Crohn's disease D. Peptic ulcer E. Whipple disease

The correct answer is C. Ten to 20% of patients who have Crohn's disease and ulcerative colitis develop an arthritis which resembles spondylitis and sacroiliitis. Similar arthropathies are seen in psoriasis or reactive arthritis syndrome (arthritis, urethritis, conjunctivitis, and rash following chlamydial infection), as well as related syndromes seen following Shigella, Salmonella, or Yersinia enterocolitis. Clinical Pearls Patients with inflammatory bowel disease (Crohn disease or ulcerative colitis) can have back pain that leads to a presentation similar to ankylosing spondylitis. With inflammatory bowel-related arthritis, there may be associated peripheral joint involvement. Patients may also have evidence of erythema nodosum or pyoderma gangrenosum. Unilateral sacroiliitis may be present. Only about 1/3 of patients will be HLA-B27-positive.

A 48-year-old secretary presents with progressive difficulty typing over the past month. She also notes that her hands begin to feel numb and weak after typing for long periods of time. On testing, which of the following deficits would be predicted? A. Difficulty in abducting the fifth finger B. Difficulty in adducting the thumb C. Difficulty in flexing digits 2 and 3 at the metacarpophalangeal joints D. Loss of sensation over the lateral half of the dorsum of the hand E. Loss of sensation over the lateral half of the palm

The correct answer is C. This is a classic presentation of carpal tunnel syndrome, which typically affects women between the ages of 40 and 60 who chronically perform repetitive tasks that involve movement of the structures that pass through the carpal tunnel. One important structure that passes through the carpal tunnel is the median nerve. Patients often note tingling, loss of sensation, or diminished sensation in the digits. There is also often a loss of coordination and strength in the thumb, because the median nerve also sends fibers to the abductor pollicis brevis, flexor pollicis brevis, and the opponens pollicis. A final function of the median nerve distal to the carpal tunnel is control of the first and second lumbricals, which function to flex digits 2 and 3 at the metacarpophalangeal joints and extend interphalangeal joints of the same digits.

A 17-year-old sexually active girl is brought to the emergency department with a 2-day history of a painful and swollen right big toe. She also has had a fever, with temperatures up to 38.9ºC (102ºF), at home for 2 days. Her parents decided to bring her to the hospital tonight as she has been unable to sleep because of the pain. On physical examination, her temperature is 101.8ºF (38.8ºC). Her first metatarsal joint of the right foot is markedly swollen and very painful to touch. An aspirated fluid from the joint reveals a WBC count of 65,000/mm3. Which of the following is the most likely diagnosis? A. Juvenile rheumatoid arthritis B. Gout C. Lyme disease D. Pseudogout E. Septic arthritis

The correct answer is E. Septic arthritis refers to microbial invasion of the synovial space. Pathogens enter the synovial space by hematogenous spread, local spread from contiguous infection, or a traumatic or surgical infection of the joint space. Accumulating fluid and pus rapidly raise the intra-articular pressure and permanently injure vessels and articular cartilage. More than 90% of cases of septic arthritis affect the joints of the lower extremity, with the knee being the most commonly involved joint. Acute septic arthritis is bacterial, with Staphylococcus aureus the most common nongonococcal organism, followed by group A streptococci and Streptococcus pneumoniae. Other organisms to consider vary by age groups. Haemophilus influenzae is common in children younger than age 3 years, but the incidence is decreasing with Hib vaccine. Neisseria gonorrhoeae may be isolated in neonates and sexually active adolescents. The teenage girl here most likely has N. gonorrhoeae. Group B beta-hemolytic Streptococcus is an important cause in neonates. Sickle cell patients are at risk for Salmonella infection. Most children who have septic arthritis appear ill and present with fever, joint pain, joint swelling, and limited movement of the affected joint. The pain is often severe and constant, with increased pain when the infected joint is moved. On physical examination, the joint is typically flexed to limit motion; local erythema, warmth, and swelling may be present. In septic arthritis of the hip, the joint is often flexed, with slight abduction and external rotation, and limitation of internal rotation with passive motion. When septic arthritis is suspected, diagnostic studies should include a complete blood count (CBC), blood cultures, an erythrocyte sedimentation rate (ESR), and plain radiographs. If the patient is febrile with an elevated ESR and CBC, the joint should be aspirated and the fluid sent for analysis. Only one third of cases will have a positive Gram stain. Up to 80% of joint fluid cultures may be positive, but the yield decreases with antibiotic use. Findings on plain radiographs may include joint space widening, increased opacity of the joint space, local soft-tissue swelling, distortion of periarticular fat or muscle shadows, and lateral displacement or dislocation of the femoral head. Treatment of septic arthritis includes surgical drainage of the purulent material from the joint and IV antibiotics. The choice of antibiotic should be based partly on the age of the patient to ensure coverage of the most likely causative organism. Empiric coverage should include an antistaphylococcal agent, either a beta-lactamase-resistant penicillin or a first-generation cephalosporin. Appropriate gram-negative coverage must be provided to neonates and adolescents. Based on presumed gonorrhea as the underlying cause of this septic joint, this patient will need to have ceftriaxone plus azithromycin for treatment. Clinical Pearls Septic arthritis is most commonly attributable to hematogenous spread. It most often presents in the knee and the hip, but may involve any joint. In infants, there can be multiple-joints involved. Arthrocentesis is usually purulent with neutrophil count >50,000/mm3 and low glucose concentration. Disseminated gonococcal infection is the most common cause of acute nontraumatic monoarthritis in young adults.

A 30-year-old woman consults a health care provider because of a painless, pea-like lesion just under her skin which is located on the back of her wrist. She finds the lesion annoying and disfiguring, and so has it removed by a surgeon. Grossly, the lesion is white and translucent and oozes gelatinous material when cut.Which of the following is the most likely diagnosis? A. Aneurysmal bone cyst B. Felon C. Ganglion cyst D. Herpetic whitlow E. Osteoid osteoma

The correct answer is C. This is a ganglion cyst. These cystic swellings occur on the hands, particularly on the dorsal aspect of the wrists, and are usually near or attached to tendon sheaths and joint capsules. About 65% of them arise near the scapholunate joint on the back of the wrist; other common sites include the distal volar aspect of the radius and the flexor tendon sheath. The lesions appear to be degenerative rather than tumorous in character, and the gelatinous material found in the center has a high hyaluronic acid content. Many ganglion cysts regress spontaneously or after needle aspiration of the contents. Recurrent ganglia or ganglia that are cosmetically unacceptable to the patient can be surgically excised, but may recur after excision. Clinical Pearls A ganglion is a cystic swelling overlying a joint or tendon sheath. Ganglia are thought to arise due to herniation of synovial tissue from a joint capsule or tendon sheath. They represent myxoid degeneration. Common locations for ganglia include the wrist and tissue adjacent to finger joints. They also occur over the dorsum of the foot and less often may arise in the knee, shoulder, spine or other intra-articular, extra-articular soft tissue, intraosseous or periosteal locations. The lesions may be unilocular or multilocular. A jelly-like fluid can be aspirated from the lesion. Surgical excision is the definitive treatment for ganglions. Ganglion cysts that are aspirated as their treatment are very likely to recur.

A 63-year-old man with a history of type 2 diabetes and hypertension comes to the health care provider after being awakened from sleep by severe pain in his right first toe. He reports the sudden onset of acute pain in the toe, rapidly followed by erythema, swelling, tenderness, and warmth. His temperature is 37.0ºC (98.6ºF), blood pressure 170/60 mm Hg, pulse 97/min, and respirations 19/min. Physical examination is normal except for swelling and severe tenderness over his metatarsophalangeal joint on the right foot. Which of the following is the most appropriate first-line treatment for this patient's condition? A. Allopurinol B. Dietary modifications C. Indomethacin D. Prednisone E. Probenecid

The correct answer is C. This patient has gout. Pain in the metatarsal phalangeal joint of his first toe is a classic symptom and is known as podagra. Common comorbid conditions include diabetes, hypertension, coronary artery disease, hyperlipidemia, and alcohol abuse. First-line treatment for gout is NSAIDs. Indomethacin is commonly used, although any nonsteroidal anti-inflammatory drug is appropriate. NSAIDs should be used with caution in patients who have underlying renal disease. Clinical Pearls The goal of therapy in an acute gout attack is prompt and safe termination of pain. While symptoms will resolve without therapy within a few days to several weeks, symptoms improve more quickly with administration of any of a broad array of anti-inflammatory drugs, with the most prompt and complete resolution when earlier treatment is introduced. Nonsteroidal anti-inflammatory drugs (NSAIDs) are first-line therapy for most patients with acute gout who have no contraindications to their use. Aspirin is usually avoided because of the paradoxical effects of salicylates on serum urate. Oral colchicine is effective for acute gout and can be diagnostic and therapeutic. For patients who cannot take NSAIDs or colchicine and who are not candidates for intra-articular corticosteroid injection because of polyarticular disease, use of oral glucocorticoids can be considered.

A 49-year-old waitress presents with painful swelling of the left knee. The symptoms began over the past 48 hours and have limited her ability to work. She has developed a fever over the past 24 hours but denies rigors, cough, rash, or headaches. On physical examination her temperature is 38.3ºC (100.9ºF), blood pressure 116/72 mm Hg, pulse 96/min, and respirations 16/min. There is a palpable effusion around the left knee capsule in association with erythema and warmth. Which of the following is the most appropriate next step in management? A. Complete blood count B. Left knee radiograph C. Left knee arthrocentesis D. Left knee MRI E. Left knee arthroscopy

The correct answer is C. This patient has monoarticular arthritis. The most likely diagnoses in this setting are crystal arthritis (gout or pseudogout) or bacterial arthritis (typically gonorrheal or staphylococcal). Establishing the diagnosis is best accomplished with arthrocentesis for examination of the extracted synovial fluid for crystals and/or bacteria. In both situations, large numbers of neutrophils may also be seen in the extracted fluid. Clinical Pearls Joint aspiration is indicated for all acute joint processes in order to analyze the joint aspirate fluid in order to guide appropriate therapy. An acutely swollen, tender, red joint may be caused by bacterial, viral, or crystal-induced disease, which is treated according to the underlying cause.

An obese 35-year-old former football player complains of joint stiffness of the knees and distal interphalangeal joints of the fingers. Physical examination reveals decreased ROM of the affected joints, crepitus, and bony swelling. Radiographs display narrowing of the affected joint spaces. Which of the following is the most likely etiology of this patient's disorder? A. Accumulation of calcium pyrophosphate in joint fluid B. Antibodies against the Fc fragment of IgG C. Chondrocyte injury and abnormal collagen activity D. Hematogenous seeding of joints during bacteremia E. Inborn error of purine metabolism

The correct answer is C. This patient has osteoarthritis, a degenerative joint disease that is very common in the elderly and in those who have a history of wear and tear or overuse of the joints (as found in football players). Chondrocyte injury occurs along with abnormal collagen activity. Predisposing factors include obesity, previous joint injury, and synovial disease. Clinically, patients have joint stiffness, decreased ROM, effusions, crepitus, and bony swelling. The most commonly affected joints include the vertebrae, hips, knees, and distal interphalangeal (DIP) joints of the fingers. (Note that Heberden nodes [terminal phalanx] and Bouchard nodes [proximal phalanx] are characteristic of this disease.) Symptoms of nerve compression may occur secondary to compression by osteophytes and bone spur development. A classic finding on radiograph is joint space narrowing. Joint fluid shows few cells and normal mucin.

A 29-year-old woman comes to the office complaining of 8 weeks of wrist and hand pain and swelling. When she wakes up in the morning, she has approximately 2 hours of stiffness and pain, which gradually improves as the day progresses. She also reports a 9-lb weight loss over the last 6 weeks along with moderate fatigue. Her temperature is 38ºC (100.4ºF), blood pressure 110/70 mm Hg, pulse 78/min and regular, and respirations 15/min. There is swelling in her proximal interphalangeal and metacarpophalangeal joints and wrists. The swelling is symmetric. There is a palpable effusion. There is no underlying liver, kidney, or bone marrow disease. An arthrocentesis shows the following results: WBC: 10,000 No crystals No organisms on Gram stain Which of the following is the most appropriate first-line pharmacologic management? A. Aspirin B. Ceftriaxone (Rocephin) C. Methotrexate (Trexall) D. Nonsteroidal anti-inflammatory drugs (NSAIDs) E. Prednisone

The correct answer is C. This patient has rheumatoid arthritis. She is complaining of symmetric joint swelling of 3 joints with >1 hour of morning stiffness. Radiographs might reveal joint erosions because it is a destructive arthritis. Synovial fluid reveals 10,000 WBCs, which is suggestive of an inflammatory arthritis. The first-line treatment for rheumatoid arthritis at presentation with mild to moderate disease is a DMARD (disease modifying agent against rheumatic disease) such as methotrexate, sulfasalazine, or leflunomide (Arava). Aggressive DMARD therapy in the setting of acute rheumatoid arthritis may help to slow the progression of the disease. Clinical Pearls The initial therapy for mild to moderate rheumatoid arthritis is DMARD therapy, because these agents can slow the course of the disease and actually positively affect the natural course of this condition. First-line DMARD therapy includes methotrexate, sulfasalazine (Azulfidine), and leflunomide (Arava). NSAIDs and corticosteroids can be used as pain mediators as a supplement to the DMARD therapy.

A 69-year-old retired police officer comes to the emergency department complaining of excruciating right foot pain. The pain began approximately 6 hours ago, immediately upon awaking. One night earlier, he had gone out for a steak dinner and drank half of a bottle of wine to celebrate his recent retirement. His past medical history is significant for hypertension, for which he takes hydrochlorothiazide. On physical examination, he is afebrile but appears acutely uncomfortable. There is swelling and tenderness in the right ankle and in the first right toe. Laboratory tests reveal a WBC count of 12,400/mm3 and a hematocrit of 39%. Serum electrolytes and liver function tests are normal. Uric acid is 4.1 mg/dL (normal 2.5-5.5 mg/dL). Which of the following will most likely be seen on examination of the joint arthrocentesis? A. Gram-negative rods B. Gram-positive cocci pairs and chains C. Negatively birefringent needle-shaped crystals D. Negatively birefringent rhomboid crystals E. Positively birefringent needle-shaped crystals

The correct answer is C. This patient has the acute onset of podagra and ankle involvement as well, secondary to gout. These attacks often follow the ingestion of red meats and/or alcohol because both of these lead to transient hyperuricemia. Note that this patient has a normal serum uric acid, which may occur during an acute attack; however, all patients who have gout have some history of hyperuricemia. After the acute gout dissipates, the goal serum uric acid level will be at 6 or lower. The appropriate evaluation of an edematous joint is an arthrocentesis. Examination of the fluid under polarizing light microscopy in a case such as this will reveal the typical uric acid crystals, which appear as negatively birefringent needle-shaped crystals (compare with choice E) under polarizing light microscopy. Clinical Pearls Joint aspiration in gout under polarizing light microscopy will reveal uric acid crystals, which appear as negatively birefringent needle-shaped crystals. Pseudogout crystals are positively birefringent and result from calcium pyrophosphate deposition disease.

A 38-year-old woman comes to the health care provider complaining of a burning discomfort in the substernal region. The symptoms are worse after meals and after reclining for sleep at night. She has a history of Raynaud phenomenon and mild hypertension. On physical examination her blood pressure is 162/94 mm Hg, pulse 78/min, and respirations 16/min. She is afebrile. There are multiple facial telangiectasias on both cheeks, and she has taut skin on both hands. The remainder of her musculoskeletal examination is unremarkable. Which of the following is most likely responsible for her chest discomfort? A. Coronary vasospasm B. Costochondritis C. Esophageal hypomotility D. Hypertension of the lower esophageal sphincter E. Pulmonary fibrosis

The correct answer is C.This patient has typical features of progressive systemic sclerosis (systemic scleroderma), as demonstrated by her Raynaud phenomenon, facial telangiectasias, taut skin, and hypertension. (If she later develops calcinosis, she will have exhibited the characteristic features of the CREST variant of progressive systemic sclerosis.) The symptoms she describes are typical of gastroesophageal reflux disease (GERD), which is seen in these patients as a result of esophageal hypomotility, as well as fibrosis of the lower esophageal sphincter (LES), which causes reduced LES pressures. The underlying disease process in progressive systemic sclerosis is a small-vessel obliteration that leads to secondary diffuse fibrosis. When this affects the esophagus, the esophageal musculature no longer contracts effectively. No specific therapy is available for the esophageal dysfunction of progressive systemic sclerosis, and patients are treated with anti-esophageal reflux regimens (e.g., antacids, H2 blockers, PPIs, frequent small feedings, and elevation of the head of bed) and periodic (mechanical) dilation of any esophageal strictures that develop. Also, the long history of reflux esophagitis will predispose for development of Barrett esophagus with the risk for progression to esophageal cancer. Clinical Pearls Progressive systemic sclerosis or scleroderma is a chronic, multisystem disease. Initial symptoms are nonspecific and include fatigue, vague musculoskeletal complaints, diffuse swelling of the hands, and Raynaud phenomena. Etiology and pathogenesis are unknown. Vascular damage and fibrosis of the lungs, heart, and GI tract may occur. Treatment goals are for disease processes that are potentially reversible (active inflammation or vasoconstriction) and minimization of functional impairment of the patient. Skin changes include tightness, thickening, and nonpitting induration, sclerodactyly, hyperpigmentation or patchy hypopigmentation, and Raynaud phenomena with at least 2 color changes (white to blue to red with rapid rewarming of the affected part of the body. Visceral changes can include bibasilar pulmonary fibrosis and bilateral reticular pattern of linear densities, lower esophageal dysphagia and dysmotility, and colonic sacculations and wide-mouthed diverticula of colon.

Intramuscular injections should be given in the upper, outer quadrant of the buttocks to prevent damage to which of the following nerves? A. Common peroneal B. Lateral femoral cutaneous C. Obturator D. Sciatic E. Superior gluteal

The correct answer is D. Injections are given in the upper, outer quadrant of the buttocks to prevent damage to the sciatic nerve, which is present in the lower quadrant. The other nerves listed are not particularly vulnerable to injections into the buttocks. Clinical Pearls Use the upper outer quadrant when giving gluteal injections because it is safest. Injury to the sciatic nerve is associated with use of the inferior dorsogluteal site for injection, because the sciatic nerve commonly courses this site.

A 13-year-old boy who has had pain in his right leg for 4 months is being evaluated in the pediatric clinic. The pain is worse at night and is unrelenting, but it can usually be relieved with ibuprofen. According to his parents, he is a physically active child and plays basketball and little league baseball. On physical examination, there is localized tenderness over the anterior aspect of the right thigh. There is also mild atrophy of the affected limb. A radiograph of the femur reveals a radiolucent nidus with surrounding reactive sclerotic bone. Which of the following is the most likely diagnosis? A. Brodie abscess B. Ewing sarcoma C. Osteosarcoma D. Osteoid osteoma E. Stress fracture

The correct answer is D. Osteoid osteoma usually affects patients between the ages of 5 and 24 years. This lesion can occur in almost any bone, although it is most commonly found in the lower extremities. Patients usually present with pain, which is characteristically worse at night and relieved with NSAIDs. The process may cause growth disturbances, such as leg length discrepancy or bowing of an extremity, or scoliosis. Radiographs usually show a sclerotic lesion and sometimes a localized lytic defect or nidus (<1 cm in diameter) within the sclerotic bone. The nidus is best demonstrated with CT. Differential diagnosis most often includes Brodie abscess or stress fracture. Histologically, the lesion consists of a nidus of immature, woven, trabecular bone and osteoblasts surrounded by dense sclerotic bone. Treatment is typically surgical excision. Medical management with NSAIDs is especially useful with lesions that are difficult or unsafe to reach (e.g., a lesion at the inner aspect of the acetabulum). Newer techniques for surgical excision include percutaneous excision with CT guidance and open surgical excision with radionuclide guidance. Pain relief after surgery is usually immediate. Clinical Pearls Osteoid osteoma usually affects patients between the ages of 5 and 24 years. It affects lower extremities and the spine most commonly. Patients present with pain that is worse at night and is relieved with NSAIDs since these growths produce a high amount of prostaglandins. It may cause leg-length discrepancy. Radiographs usually show a sclerotic lesion and sometimes a localized lytic defect or nidus within the sclerotic bone. Treatment is surgical excision.

A 13-year-old boy is seen for right knee pain that has persisted for the past 2 months despite the use of over-the-counter analgesics. His mother states that he has been limping since he started to have the pain. The pain is of insidious onset, but its intensity sometimes suddenly increases. On physical examination the boy is obese; his weight is higher than the 90th percentile. Flexion of the right hip causes marked outward rotation and reproduces the pain. Internal rotation of the right hip is limited, and the right leg is slightly shortened compared to the left. Which of the following is the most likely diagnosis? A. Juvenile rheumatoid arthritis B. Legg-Calve-Perthes disease C. Septic arthritis D. Slipped capital femoral epiphysis (SCFE) E. Toxic synovitis

The correct answer is D. Slipped capital femoral epiphysis (SCFE) is a relatively common hip problem, usually seen in the adolescent near the time of peak growth velocity. It is characterized by displacement of the femoral epiphysis from the femoral neck through the growth plate. SCFE occurs more commonly in younger children, and boys are affected more often than girls. Mechanical stress during the growth spurt is an important cause and at least half of affected children are obese. Because of SCFE's frequency, risk for degenerative arthritis, and possible sudden instability and vascular damage, early detection is extremely important. Clinical findings can vary depending on whether the slip is stable. In a stable slip, an obese adolescent boy may complain of activity-related hip pain that is relieved by rest. There may be a mild limp after strenuous activity. Physical examination shows pain with hip motion, restriction of flexion, and particularly, internal rotation while the hip is flexed to 90 degrees. The diagnosis can be elusive, because referral of the pain may be to the distal thigh, presenting as knee pain. All children who complain of knee pain should have their hip examined. An unstable slip does not allow the child to bear weight, even with crutches. This should be considered the equivalent of an acute, displaced fracture of the hip and should be treated urgently. The child is in extreme pain and usually gives a history of a fall from a bicycle or a tree; however, the fall may also be trivial if the epiphysis had pre-existing weakness. Radiographs confirm the diagnosis, and the patient must be urgently admitted to the hospital for treatment. The risk for avascular necrosis, even with immediate treatment, is high. Radiographs are obtained with the patient in the supine position for anteroposterior (AP) and frog lateral pelvis radiographs. Once the diagnosis of a stable slipped epiphysis is made in a walking child, surgery should be performed without delay. The most common procedure is percutaneous pin or screw fixation between the femoral neck and the center of the epiphysis. Clinical Pearls Slipped capital femoral epiphysis (SCFE) is characterized by a displacement of the capital femoral epiphysis from the femoral neck through the physial plate. This condition occurs more frequently in males at the time of their peak growth. Obesity is a significant risk factor. This condition presents with pain (can be knee rather than hip) and altered gait. The classic presentation is an obese adolescent with a complaint of nonradiating, dull, aching pain in the hip, groin, thigh, or knee and no history of preceding trauma. The pain is increased by physical activity and may be chronic or intermittent. The diagnosis of SCFE usually can be made on plain radiographs, which reveal an apparent posterior displacement of the femoral epiphysis, like ice cream slipping off a cone. The treatment of SCFE is operative. Children with SCFE should be referred promptly to an orthopedic surgeon; they must avoid bearing weight until they have undergone orthopedic evaluation. The complications of SCFE include osteonecrosis of the femoral head and chondrolysis, both of which increase the risk of subsequent development of osteoarthritis. The prognosis of SCFE is related to the severity of the slip.

A 34-year-old woman presents with fatigue, malaise, and swollen, tender joints. Physical examination is significant for a maculopapular eruption over sun-exposed areas, including the face. Examination of a peripheral blood smear reveals mild thrombocytopenia. Which of the following autoantibodies, if present, would be most specific for the diagnosis of the patient's disorder? A. Anti-centromere antibody B. Anti-IgG antibody C. Antinuclear antibody D. Anti-double stranded DNA antibody E. Anti-SS-A (Ro) antibody

The correct answer is D. The patient described probably has systemic lupus erythematosus (SLE), which often presents with fatigue, malaise, fever, GI symptoms, arthralgias, and myalgias. Hematologic abnormalities include anemia, leukopenia, lymphocytopenia, and thrombocytopenia. A circulating anticoagulant may prolong the activated partial thromboplastin time (APTT). Cutaneous manifestations include a malar rash and a generalized maculopapular eruption, both of which are photosensitive. Antibodies to the Smith antigen (core proteins of small ribonucleoproteins found in the nucleus) are present in only 20 to 30% of patients who have SLE, but are quite specific for the disease, occurring only rarely in other autoimmune diseases. Anti-ds DNA is a specific finding that occurs with systemic lupus erythematosus (SLE). Clinical Pearls The screening test for SLE is the ANA (antinuclear antibody), which reflects that the SLE patient has immune dysregulation. Specific laboratory findings for SLE include the Smith antibody and double-stranded DNA antibody. Both of these tests are used as confirmatory tests for SLE.

A 13-year-old obese boy complains of persistent knee pain for several weeks. The family brings him in because he has been limping. He sits on the examining table with the sole of the foot on the affected side pointing to the other leg. Physical examination is normal for the knee, but shows limited hip motion. As the hip is flexed, the leg goes into external rotation and cannot be rotated internally. Which of the following is the most likely diagnosis? A. Avascular necrosis of the femoral head B. Developmental dysplasia of the hip C. Osteogenic sarcoma of the lower femur D. Slipped capital femoral epiphysis E. Tibial torsion with foot inversion

The correct answer is D. Slipped capital femoral epiphysis is an orthopedic emergency. The clinical picture is classic: an obese boy in his early teens who is limping and cannot rotate his leg internally. Part of the classic presentation is also the fact that hip pathology often produces knee pain, but the knee is normal on physical examination. Clinical Pearls Slipped capital femoral epiphysis (SCFE) is characterized by a displacement of the capital femoral epiphysis from the femoral neck through the physial plate. This condition occurs more frequently in males at the time of their peak growth. Obesity is a significant risk factor. This condition presents with pain (can be knee rather than hip) and altered gait. The classic presentation is an obese adolescent with a complaint of nonradiating, dull, aching pain in the hip, groin, thigh, or knee and no history of preceding trauma. The pain is increased by physical activity and may be chronic or intermittent. The diagnosis of SCFE usually can be made on plain radiographs, which reveal an apparent posterior displacement of the femoral epiphysis, like ice cream slipping off a cone. The treatment of SCFE is operative. Children with SCFE should be referred promptly to an orthopedic surgeon; they must avoid bearing weight until they have undergone orthopedic evaluation. The complications of SCFE include osteonecrosis of the femoral head and chondrolysis, both of which increase the risk of subsequent development of osteoarthritis. The prognosis of SCFE is related to the severity of the slip.

A 23-year-old construction worker presents to the emergency department after falling down on his outstretched right hand. A radiograph reveals a fracture of one of the wrist bones. The health care provider is concerned about the risk for avascular necrosis. Which of the following bones was fractured? A. Capitate B. Lunate C. Pisiform D. Scaphoid E. Trapezoid

The correct answer is D. Suspect fracture of the scaphoid (navicular) bone in any young adult who has fallen on an outstretched hand (classic clue). The health care provider was concerned about the risk for avascular necrosis because the blood supply of the bone is located distally, and a fracture will deprive the proximal region of the bone of its arterial nourishment. Osteoarthritis and avascular necrosis are complications of scaphoid fractures that do not heal properly.

A 45-year-old woman who repeatedly wears high-heeled, pointed shoes complains of pain in the forefoot after prolonged standing or walking. Occasionally she also experiences numbness, a burning sensation, and tingling in the metatarsal head area. Physical examination shows no obvious deformities and a very tender spot in the third interspace, between the third and fourth toes. Pain is worse when the heads of the metatarsals are compressed together during the physical examination. There is no redness, limitation of motion, or signs of inflammation. Which of the following is the most likely diagnosis? A. Gout B. Hallux rigidus C. Metatarsophalangeal articulation pain D. Morton neuroma E. Plantar fasciitis

The correct answer is D. The location and circumstances are classic for Morton neuroma, a benign neuroma of the third plantar interdigital nerve. Tight toe boxes in shoes make this condition worse. Gout (choice A) usually happens to obese, elderly males, and redness and signs of inflammation in the affected joint are evident. Clinical Pearls Morton neuroma is a painful interdigital nerve typically located in the foot's third interspace, between the third and fourth toes. Diagnosis is made at examination by eliciting exquisite pain on palpation of the mentioned area. Pain is worse when the metatarsal heads are pressed together. Conservative treatment includes avoidance of wearing tight-fitting shoes. Metatarsal support pads and padded shoe supports can also be used. Surgical intervention involves removal of the neuroma. Surgery is pursued for disabling symptoms that do not respond to conservative measures.

After a grand mal seizure, a 32-year-old epileptic woman notices pain in her right shoulder and she cannot move it. She goes to a minor emergency clinic, where she has a limited physical examination and anteroposterior (AP) radiographs of her shoulder. The films are read as negative, and she is diagnosed as having a sprain and given pain medication. The next day, she still has the same pain and is unable to move her arm. She comes to the emergency department holding her arm close to her body, with her hand resting on her anterior chest wall. Which of the following is the most likely diagnosis? A. Acromioclavicular separation B. Anterior dislocation of the shoulder C. Articular cartilage crushing D. Posterior dislocation of the shoulder E. Torn teres major and minor muscles

The correct answer is D. The mechanism of injury (massive contraction of all muscles in the area) and the missed diagnosis on a single view AP radiograph are classic for posterior dislocation of the shoulder. The highest risk injuries leading to posterior shoulder dislocation are generalized seizure, electrocution with falling facedown, and football linemen who are hit directly on the anterior surface of the shoulder when they are attempting to block. Axillary or Y-view radiographs are needed to make the diagnosis. Clinical Pearls Massive muscular contraction, as in a seizure, can produce a posterior dislocation of the shoulder. The patient will present holding the arm close to the body with the hand resting on the chest wall. Axillary-view radiographs are needed to make the diagnosis.

A 27-year-old front-seat passenger in a car involved in a head-on collision relates that he hit the dashboard with his knees. He is specifically complaining of severe pain in his right hip, rather than knee pain. He lies in the stretcher in the emergency department with the right lower extremity shortened, adducted, and internally rotated. Which of the following is the most likely injury? A. Femoral neck fracture B. Fracture of the shaft of the femur C. Intertrochanteric fracture D. Posterior dislocation of the hip E. Posterior dislocation of the knee

The correct answer is D. The mechanism of injury is classic. As the knee hits the dashboard in the sitting position, the femoral head is driven backward and out of the socket. The position of the injured extremity is also typical, with the internal rotation produced by the posteriorly dislocated femoral head. This injury is an orthopedic emergency because of the tenuous blood supply of the femoral head. Posterior hip dislocations can be treated with closed reduction if done soon after the hip dislocation. These hip dislocations may have fractures of the posterior rim of the acetabulum so imaging is necessary to rule out any associated fracture. Clinical Pearls High-energy, direct blunt force trauma is the most common cause of hip dislocations. Motor vehicle collisions in which the knee impacts the dashboard are common mechanisms of injury. Prosthetic hip joints may dislocate with much less force. Dislocation of the hip joint is an orthopedic emergency. Timely diagnosis and management may minimize the significant morbidity that may result.

A 6-month-old boy is found to have bilateral metaphyseal fractures of both proximal and distal ends of the tibia. The mother says that her boyfriend takes care of the infant while she is at work. Her boyfriend explains the injuries as the result of multiple falls from bed. Which of the following is the most likely diagnosis? A. Accidental trauma B. Osteogenesis imperfecta C. Osteopetrosis D. Physical abuse E. Rickets

The correct answer is D. There are specific types of traumatic injuries that are virtually pathognomonic of child abuse and cannot be explained by accidental trauma (choice A). Metaphyseal fractures are the most typical (although not the most frequent) injuries caused by physical abuse. They are produced by pulling and twisting the limbs, which results in traction on the metaphyses transmitted by the joint ligaments. Other skeletal injuries that should suggest abuse include multiple fractures of different stages of healing and fractures of posterior ribs, scapula, spinous processes, and sternum. The following is a list of injury patterns typically associated with child abuse: Bruises on the buttocks, abdominal wall, genitalia, back, and dorsal hands and feet. Accidental bruises in the active toddler are usually on bony prominences. Head injuries: subdural hematoma, diffuse axonal injury, and retinal petechiae Abdominal injuries: intestinal rupture or hematoma occurs more frequently than injuries to solid organs from a hard blow. Burns: cigarette burns and burns on the buttock area caused by scalding water. Clinical Pearls Health care providers have a duty to protect patients under their care. Physical abuse should be suspected for injuries that do not have a reasonable explanation for their occurrence or based upon injury patterns that are not consistent with the child's stage of development. Features of the history that are suspicious for inflicted injury include: Inconsistencies and/or discrepancies in caretakers' accounts of the circumstances surrounding the injury Unwitnessed injuries Injuries attributed to the patient's siblings Injuries inconsistent with the child's developmental stage Injuries inconsistent with the mechanism of injury offered Heath care providers are mandated reporters of suspected child or elder abuse, meaning that they have a duty to report abuse injuries when they are suspected so that further harm cannot continue. Health care providers do not have to have proof of the abuse but they should report suspected child abuse to appropriate authorities.

An elderly woman with osteoporosis is taken to the emergency department following a fall. One of her lower extremities appears shortened and is externally rotated. A fracture of which part of the femur is suggested by these findings? A. Greater trochanter B. Lateral epicondyle C. Medial epicondyle D. Neck E. Shaft

The correct answer is D. This is a classic presentation of a fracture of the neck of the femur. This type of fracture typically occurs in postmenopausal women who have significant bone resorption caused by osteoporosis. Dislocation of the head of the femur can produce a similar effect. The change in the position of the leg is caused by the action of the gluteal muscles, particularly the gluteus maximus pulling on the proximal fracture site resulting in external rotation and shortening. Clinical Pearls Osteoporosis results in a decreased bone density and abnormal bone architecture that weakens the structural support of the affected bone. Patients who have osteoporosis are most likely to sustain hip and vertebral fractures, and they are also at risk for wrist fractures. Femoral neck fractures are associated with osteoporosis and these fractures are most likely to result in external rotation and shortening of the affected lower extremity. A vascular necrosis is a known complication with femoral neck fractures if they are displaced, because of the potential for blood supply to be interrupted.

A 14-year-old boy presents with a 1-month history of knee pain and a 6-pound weight loss. He is pale and afebrile. A radiograph reveals a densely sclerotic lesion in the distal femur extending from the growth plate into the diaphysis. The periosteum is lifted, forming an angle with the cortex. The surrounding soft tissue resembles a sunburst on the radiograph. Which of the following is the most likely diagnosis? A. Nonossifying fibroma B. Osteochondroma C. Osteomyelitis D. Osteosarcoma E. Paget disease

The correct answer is D. This patient has osteosarcoma, a malignant bone tumor that produces osteoid and bone. Prognosis is typically poor. Radiograph reveals bone destruction, soft tissue with sunburst appearance, and Codman triangle (periosteal elevation that forms an angle with the cortex of the bone), all classic clues to the diagnosis. Other hints were the patient's weight loss and pallor, which should have raised suspicion that a malignancy existed. Clinical Pearls Osteosarcomas are uncommon primary malignant tumors of bone that are characterized by the production of osteoid or immature bone by the malignant cells. The age distribution of osteosarcoma incidence is bimodal, with peaks in early adolescence and again in adults over the age of 65. Osteosarcoma is the most common primary bone tumor affecting children and young adults; the peak age is between 13 and 16.

A 16-year-old boy comes to the health care provider complaining of a persistent sunburn on his face that started when he was on vacation 2 weeks earlier and does not seem to be improving. He also complains of headache, chills, fever, and joint pains that are most prominent in the hands with the pain being the worst in the morning. Aspirin seems to improve the symptoms somewhat. He has no significant past medical history and does not take any medication on a regular basis. He denies allergies to medications. On physical examination, the patient is a well-developed and well-nourished young man. His temperature is 37.3ºC (99.1ºF), blood pressure is 140/90 mm Hg, pulse is 80/min, and respirations are 16/min. Inspection of the skin reveals diffuse erythema and edema of the nose and cheeks in a butterfly distribution and erythema and edema of the skin overlying the interphalangeal part of his fingers. The proximal and distal interphalangeal joints of the fingers are edematous and tender and his fingertips are studded with minute violaceous papules tender to touch. On the hard palate, there are several shallow ulcers up to 1 cm in diameter. Laboratory studies show: RBCs 2.8 x 1012/L (normal 4.7-6.1 million cells) WBCs 2.2 x 109/L (normal 5,000-10,000) Platelets 200 x 109/L (normal 150,000-400,000) Erythrocyte sedimentation rate 57 mm/h (normal <20 mm/h) Urine protein 4+ (normal negative) Which of the following is the most appropriate screening lab test for this patient? A. Anticentromere antibodies B. Antihistone antibodies C. Anti-Jo-1 antibodies D. Antinuclear antibody E. Anti-Ro antibodies

The correct answer is D. This patient has systemic lupus erythematosus, an autoimmune inflammatory rheumatoid disease affecting multiple organ systems. The etiology is unknown, but it is believed to be multifactorial. Autoantibodies are produced against self-antigens, especially DNA and other nuclear antigens. Sunlight and some infections can exacerbate episodes of lupus. Antinuclear antibody test is the best screening laboratory study. Antinuclear antibody testing is so sensitive that if the test is negative, a search for another disease should be made. Antibodies to double-stranded DNA are much more specific and are present during active disease. C3, C4, and CH50 are all decreased in active disease. Anti-Smith antibodies do not measure disease activity, but are fairly specific for systemic lupus erythematosus. Renal biopsy confirms lupus nephritis. Clinical Pearls Systemic lupus erythematosus is an autoimmune disorder that can affect any system in the body. Women of childbearing age are most commonly affected. Antinuclear antibody formation is characteristic of this disorder and this is the initial test that should be performed when this disorder is suspected. ANAs are IgG antibodies. Confirmatory tests include double-stranded DNA and identification of the Smith antigen.

A 74-year-old woman is one day postoperative left total knee replacement for severe osteoarthritis (OA). She has a past medical history significant for type 1 diabetes mellitus and glaucoma. She continues to take daily NPH insulin and has good control of her blood glucose. She also takes oxycodone, which was given to her for pain. She is planning on getting involved in a physical therapy rehabilitation program at the local hospital. On review of her medications, which of the following is most acutely indicated at this time? A. Nonsteroidal anti-inflammatory drug (NSAID) B. ACE inhibitor C. Aspirin D. Low-molecular-weight heparin E. Unfractionated heparin

The correct answer is D. This patient is post-total knee replacement and is not on anticoagulation therapy. Her risk for developing deep venous thrombosis (DVT) and subsequent pulmonary embolism is very high. An anticoagulant such as low-molecular-weight heparin or unfractionated heparin should be initiated. Low-molecular-weight heparin has proven more effective than unfractionated heparin for thromboprophylaxis following total knee replacement. Other treatments that can be done are the direct thrombin inhibitors such as rivaroxaban (Xarelto) for at least five days postoperatively followed by aspirin therapy. Clinical Pearls Low-molecular-weight heparin is the treatment of choice for the prophylaxis of DVT in postsurgical patients. It may still be used in renal disease, provided that renal dosing adjustments are made. Patients are now able to transition to the Factor Xa inhibitor rivaroxaban (Xarelto) and aspirin five days later as this treatment protocol has been recently been identified as being effective in preventing DVT following joint replacement.

A 34-year-old man has a swollen left knee of 2 days' duration. He denies any known trauma to that region and has no prior history of any musculoskeletal complaints. He is in otherwise excellent health. He is homosexual and practices safe sex with a single partner. On physical examination the knee is swollen, tender to palpation, erythematous, and has a limited ROM. An arthrocentesis is performed. Which of the following is most suggestive of a septic arthritis in this patient? A. A complete blood cell count with 14,300 WBCs per mL B. A joint fluid aspirate with a WBC count of 28,000 per mL C. A joint fluid aspirate with a WBC count of 36,000 per mL D. A joint fluid aspirate with a WBC count of 48,000 per mL E. A joint fluid aspirate with a WBC count of 93,000 per mL

The correct answer is E. Septic arthritis will produce the highest joint fluid WBC counts, typically with counts >50,000 per mL. Noninflammatory arthritis such as osteoarthritis typically produces counts <10,000 per mL. If septic arthritis is suspected based on the aspirate WBC count, then antibiotics should be started while awaiting cultures. Failure to do so until after the organism has been identified could lead to irreversible joint destruction. Clinical Pearls Patients who have a septic joint may have excessively high WBC counts in the joint aspirate, but an absence of increased WBC in the joint aspirate does not rule out a septic joint. That is because only 50% of patients with a septic joint have increased WBC count during aspiration. Gram stain, culture, and microscopic examination of the joint aspirate fluid are routinely performed in order to support the diagnosis for determining the etiology for the inflamed, hot joint. Virtually any microbial pathogen is capable of causing bacterial arthritis. Organisms such as S. aureus and Streptococci, however, have a higher propensity to cause joint infections than gram-negative bacilli, which typically only produce these infections after trauma or in patients who have severe underlying immunosuppression. Patients who have bacterial (nongonococcal) arthritis present acutely with a single swollen and painful joint (i.e., monoarticular arthritis). The knee is involved in more than 50% of cases, but wrists, ankles, and hips are commonly infected. Bacterial arthritis is part of the differential diagnosis of acute monoarthritis.

A patient with systemic lupus erythematosus (SLE) very much wants to become pregnant. What should her health care provider tell her regarding pregnancy? A. There is no increased risk to the fetus B. There is an increased risk for cardiovascular malformations C. There is an increased risk for nervous system malformations D. There is an increased risk for renal malformations E. There is an increased risk for spontaneous abortions and prematurity

The correct answer is E. Systemic lupus erythematosus (SLE) predominantly affects younger women, and so the question of lupus and pregnancy may arise frequently in clinical practice. Patients who have SLE have an increased incidence of spontaneous abortion, fetal death in utero, and prematurity. The mother may experience an exacerbation in the activity of her disease in the third trimester or peripartum period, and it may be difficult to distinguish between active SLE and preeclampsia. Therapy of pregnant patients who have SLE is problematic, and these women should have their prenatal care provided by a specialist. Clinical Pearls Patients who have SLE may develop antiphospholipid antibodies, which is associated with high rates of fetal loss. Even without the presence of antiphospholipid antibodies, women who have SLE have a higher rate of fetal loss than patients who do not have SLE.

A 4-year-old boy is being evaluated for short stature. He has a history of multiple bone fractures in the past. He requires a wheelchair to ambulate and has hearing difficulty. On physical examination, his height is below the 5th percentile. His sclerae are blue in color. There is marked deformity of his lower extremities. Which of the following is the most likely diagnosis? A. Achondroplasia B. Constitutional delay of growth C. Developmental dysplasia of the hip D. Familial short stature E. Osteogenesis imperfecta

The correct answer is E. The boy as osteogenesis imperfecta, a group of disorders caused by deficiency in the synthesis of type 1 collagen. The hallmark of the disease is prominent skeletal deformity, though other anatomic structures rich in type 1 collagen are affected as well (i.e., joints, eyes, ears, skin, and teeth). The clinical expression of osteogenesis imperfecta constitutes a spectrum of disorders, all marked by extreme skeletal fragility. Four major subtypes have been distinguished. Type I is more often caused by an acquired rather than a hereditary mutation. It permits a normal life span but with an increased number of fractures during childhood. These decrease in frequency after puberty. Other findings include blue sclerae caused by a decrease in collagen content, making the sclera translucent and allowing partial visualization of the underlying choroid; hearing loss related to both a sensorineural deficit and impeded conduction owing to abnormalities in the bones of the middle and inner ear; and dental imperfections (small, misshapen, and blue-yellow teeth) secondary to a deficiency of dentin. Type II is at one end of the spectrum and is uniformly fatal in utero or during the perinatal period. It is characterized by extraordinary bone fragility with multiple fractures occurring when the fetus is still within the womb. Type III: The bony deformity is pronounced and not necessarily caused by fractures. Mobility is impaired, and most patients require a wheelchair at an early age. Stature may be severely compromised. Because of a progressive vertebral column deformity and rib fractures, restrictive lung disease is a common problem. A basilar impression causing compression of the brainstem and the craniocervical junction can produce central sleep apnea, headache, and upper motor neuron signs. Type IV: Patients generally have reduced stature, some bony deformity, and abnormal teeth that are opalescent and wear easily. The tendency to fracture is highest in childhood and lessens with adolescence. A distinguishing characteristic of type IV is a normal scleral hue. The management of skeletal complications largely depends on orthopedic, physical, and occupational therapy approaches. The long-term goal is to maintain function and independence as an individual. These goals can be advanced in some by judicious use of intramedullary rods in the long bones of the legs. If mobility, especially ambulation, can be maintained, the demineralization associated with inactivity can be avoided. Unaffected parents of a child with osteogenesis imperfecta, as well as all affected individuals, should have genetic counseling. Clinical Pearls Osteogenesis imperfecta (OI) is a rare inherited connective tissue disorder with many phenotypic presentations. Severely affected patients suffer multiple fractures with minimal or no trauma, and infants with the worst form of OI die in the perinatal period. Mild forms of OI may be manifested by only premature osteoporosis or severe postmenopausal bone mineral loss. Type I collagen is an important structural protein for bone, tendon, ligament, skin, and sclerae. OI is most commonly caused by mutations in genes encoding the alpha-1 and alpha-2 chains of type I collagen or proteins involved in posttranslational modification of type I collagen.

Several months after sustaining a crushing injury to his arm, a patient complains bitterly about constant, burning, agonizing pain in that arm that does not respond to the usual analgesic medications. The pain in his arm is aggravated by the slightest stimulation of the area, such as rubbing from the shirtsleeves. The arm is cold, cyanotic, and moist, but it is not swollen. Pulses at the wrist are normal, and neurologic function of the 3 major nerves is intact. Which of the following is most appropriate interventions will provide diagnostic confirmation and therapeutic treatment for this condition? A. Angiogram and subclavian vein bypass B. Cervical spine radiographs and cervical rib resection C. Doppler studies and arterial reconstruction D. Doppler studies and fasciotomy E. Sympathetic block and surgical sympathectomy

The correct answer is E. The description is that of complex regional pain syndrome, formerly known as reflex sympathetic dystrophy. The sympathetic nerve block, if done early in the course of the condition, can alleviate pain during physical therapy. The primary goal of physical therapy is to reduce pain and increase limb function and mobility. The goals of physical therapy are to reduce edema, decrease muscle guarding, and improve function in everyday activities. If sympathetic block relieves the symptoms, permanent cure can also be obtained with surgical sympathectomy. Clinical Pearls Complex regional pain syndrome, formerly known as reflex sympathetic dystrophy or causalgia, is described as an intense burning pain after an injury. Pain is out of proportion to acute injury. An early aggressive approach is preferred, because it commonly progresses. Pain management with local infiltrations of anesthetics or with nerve sections is usually needed. Physical therapy is the modality treatment of choice and is used to improve ROM, decrease edema, and improve function in daily activities. Allodynia and hyperalgesia are typically universally present.

A 39-year-old dental hygienist comes to the health care provider with the complaint of swelling in the right knee. Over the past 24 hours, her right knee suddenly became swollen and painful to weight bearing along with a limited ROM. She does not recall any history of arthritis or trauma in that region. She is not aware of any needle-stick injuries while at work. Until the onset of knee pain, she had been exercising daily on a treadmill. She has not been to a health care provider in 15 years. Physical examination reveals tenderness, swelling, and erythema in the right knee. There is painless flexion and limitation in extension. Which additional information would be most relevant in this patient's history? A. Family history of rheumatoid arthritis B. History of bacterial gastroenteritis C. History of hepatitis B vaccination D. History of a traumatic right ankle injury E. Sexual history

The correct answer is E. This patient has monoarticular arthritis of the right knee. The major differential diagnosis in this condition is a crystalline arthritis (gout) versus an infectious arthritis (Staphylococcus vs. Gonococcus). Knowledge of this patient's sexual history, which could put her at risk for gonococcal infection, would be relevant. The etiology for this acutely swollen knee can then be identified via joint aspiration.

A 72-year-old man with a history of obesity and hypertension treated with nifedipine comes to the health care provider complaining of worsening right-sided knee pain. He has suffered from this knee pain for 10 years, but it is no longer relieved with NSAIDs as it once was. The knee pain gets progressively worse as the day goes on and it improves with rest. When asked about other joint involvement, he reports that he has less severe pain in his right wrist. His temperature is 37ºC (98.6ºF), blood pressure 160/90 mm Hg, pulse 68/min, and respiration rate 25/min. Physical examination reveals on obese man in no acute distress. His knee is not swollen or warm but crepitus can be appreciated. His left knee and right wrist are normal. Which of the following is consistent with this patient's history? A. Antinuclear antibody positive B. Elevated erythrocyte sedimentation rate C. Joint erosions on radiograph D. Rheumatoid factor positive E. Unequal joint spaces on radiograph

The correct answer is E. This patient has osteoarthritis (OA). Clues to this diagnosis include the patient's risk factors, such as advanced age and obesity. The distribution of joints is also typical, with the weight-bearing joints such as the hip and knee being commonly affected together with the small joints of the fingers and base of the thumb. Asymmetric joint involvement is common. Morning stiffness is seen but resolves in a short period of time, as opposed to that of rheumatoid arthritis (RA). The more typical complaint is worsening joint pain with activity. Crepitus can be appreciated on physical examination of the involved joint. The common radiographic finding is unequal joint spaces with osteophyte formation. Clinical Pearls Osteoarthritis is common in older, obese patients and involves the weight-bearing joints, such as the knee and hip. Osteophytes and uneven joint spaces are characteristic on radiograph. A common physical finding is crepitus, described as a "crunching" feeling on palpation with simultaneous movement of the joint.

A middle-aged homeless man is brought to the emergency department because of very severe pain in his forearm. He had passed out after drinking a bottle of cheap wine, and then slept on a park bench for an indeterminate time, probably more than 12 hours. Shortly after he woke up and began to move around, the forearm pain began. There are no signs of trauma, but the muscles in his forearm are very firm and tender to palpation. Passive motion of the fingers and wrist elicits excruciating pain. Pulses at the wrist are normal.Which of the following is the most appropriate next step in management? A. Analgesics and observation B. Immobilization in a sling C. Immobilization in a plaster cast D. Emergency embolectomy E. Emergency fasciotomy

The correct answer is E. This presentation is classic for compartment syndrome, triggered by prolonged ischemia due to the position of the arm and forearm while the patient slept. After the patient's arm and forearm was ischemic, there was reperfusion to these ischemic tissues with resultant swelling in this closed compartment. The 2 most common sites of compression syndrome are the forearm and lower leg. This patient has the most reliable physical finding (pain on passive extension), and the diagnosis is not ruled out by normal pulses. Only a fasciotomy will solve his problem. Failure to relive the increased intracompartmental pressures in the forearm may result in permanent tissue necrosis. Clinical Pearls Acute compartment syndrome (ACS) occurs when increased pressure within a compartment bounded by unyielding fascial membranes compromises the circulation and function of the tissues within that space. ACS is a surgical emergency. ACS can result from trauma, ischemia-reperfusion injury, coagulopathy, certain animal envenomations and bites, extravasation of IV fluids, injection of recreational drugs, and prolonged limb compression. Early symptoms of ACS include progressive pain out of proportion to the injury; signs include tense swollen compartments. The initial sign is pain with passive stretching of muscles within the affected compartment. Immediate management of suspected ACS includes relieving all external pressure on the compartment. Any dressing, splint, cast, or other restrictive covering should be removed. The limb should NOT be elevated. Analgesics should be given and supplementary oxygen provided. Hypotension reduces perfusion and should be treated with intravenous isotonic saline. Fasciotomy to fully decompress all involved compartments is the definitive treatment for ACS. Delays in performing fasciotomy increase tissue loss and morbidity.

A 38-year-old woman comes to the office complaining of a new-onset rash on her face, hair loss, malaise, and fatigue that has been worsening over the previous month. She has also noticed that sun exposure makes her rash flare to a great degree. Her medical history is significant for food allergies and acute thyroiditis as a teenager. Her only medication is a daily thyroid hormone supplement. On physical examination, there is ill-defined erythema and edema on the nose and cheeks. There is diffuse thinning of the scalp hair, with short hairs on the anterior hairline. The fingertips are studded with minute, violaceous papules that are tender to touch. Urinalysis shows 4+ protein. Laboratory studies show: RBC 2.9 WBC 2.0 Platelets 220 ESR 57 Anti-nuclear 1:160 Which of the following antibodies is the most specific serologic test for this disease? A. Anticentromere antibodies B. Antihistone antibodies C. Anti-Jo-1 antibodies D. Anti-Ro antibodies E. Anti-Smith antibodies

The correct answer is E. Young to middle-aged women are predominantly affected with systemic lupus erythematosus, showing a wide range of signs and symptoms. Skin involvement occurs in 80% of cases and is likely to be very helpful in making the correct diagnosis. Of all the serologic tests, anti-Smith antibodies carry the highest specificity for lupus erythematosus. Clinical Pearls The screening test for SLE is the antinuclear antibody, which is seen in practically all patients who have SLE. Specific tests that are used as confirmatory tests include double-stranded DNA and anti-Smith antibodies.


Set pelajaran terkait

Anatomy and Physiology 1st Final (corrected)

View Set

Chapter 11 - Skin, Hair, and Nails Assessment, Chapter 12 - Head and Neck, including Lymph Nodes and Modules, Chapter 15 - Nose, Sinuses, Mouth, and Throat, Chapter 17 - Heart and Neck Vessels Assessment, Chapter 18 - Peripheral Vascular and Lymphati...

View Set

Chapter 12: Corporate Governance and Business Ethics

View Set

Quiz 08 - Coffee and Measurements

View Set

Bus&201 Final Study Set Chapter 18

View Set

Global Ch. 1, Chapter 2, International Marketing Ch. 2, Int'l Marketing - chapter 2, Mktg 452 Chapter 1

View Set